Exam 3 Practice

अब Quizwiz के साथ अपने होमवर्क और परीक्षाओं को एस करें!

16. When measuring the BP to ensure consistency and to facilitate early detection of BP changes consistent with gestational hypertension, the nurse should: a. place the woman in a supine position. b. allow the woman to rest for at least 15 minutes before measuring her BP. c. use the same arm for each BP measurement. d. use a proper sized cuff that covers at least 50% of her upper arm.

16. c; the woman should be seated or in a lateral position, she should rest for 5 to 10 minutes, and the cuff should cover 80% of the upper arm.

17. When caring for a woman with mild preeclampsia, it is critical that during assessment the nurse is alert for signs of progress to severe preeclampsia. Progress to severe preeclampsia would be indicated by which one of the following assessment findings? a. Proteinuria of 31 or greater b. Platelet level of 200,000/mm3 c. Deep tendon reflexes 21, ankle clonus is absent d. BP of 154/94 and 156/100, 6 hours apart

17. a; with severe preeclampsia, the DTRs would be more than 31 with possible ankle clonus; the BP would be more than 160/110; thrombocytopenia with a platelet level less than150,000 mm3.

18. A woman's preeclampsia has advanced to the severe stage. She is admitted to the hospital and her primary health care provider has ordered an infusion of magnesium sulfate be started. In fulfilling this order the nurse would implement which of the following? (Circle all that apply.) a. Prepare a loading dose of 2 g of magnesium sulfate in 200 ml of 5% glucose in water to be given over 15 minutes. b. Prepare the maintenance solution by mixing 40 g of magnesium sulfate in 1000 ml of lactated Ringer's solution. c. Monitor maternal vital signs, fetal heart rate (FHR) patterns, and uterine contractions every 2 hours. d. Expect the maintenance dose to be approximately 2 g/hour. e. Report a respiratory rate of 14 breaths or less per minute to the primary health care provider immediately. f. Recognize that urinary output should be at least 25-30 ml per hour.

18. b, d, and f; the loading dose should be an IV of 4 to 6 g diluted in 100 mL of intravenous fluid; maternal assessment should occur every 15 to 30 minutes and FHR and UC continuously; respirations should be less than 12

19. The primary expected outcome for care associated with the administration of magnesium sulfate would be met if the woman exhibits which of the following? a. Exhibits a decrease in both systolic and diastolic BP b. Experiences no seizures c. States that she feels more relaxed and calm d. Urinates more frequently, resulting in a decrease in pathologic edema

19. b; magnesium sulfate is a CNS depressant given to prevent seizures.

A hematoma is the collection of blood beneath the intact skin layer following an injury to a blood vessel. T/F

ANS: True A hematoma is a localized collection of blood in connective or soft tissue under the skin that follows injury of or laceration to a blood vessel without injury to the overlying tissue. At the time of injury, pressure necrosis and inadequate hemostasis occur.

A woman has been diagnosed with mild preeclampsia and will be treated at home. The nurse, in teaching this woman about her treatment regimen for mild preeclampsia, would tell her to do which of the following? (Circle all that apply.) a. Check her respirations before and after taking her oral dose of magnesium sulfate b. Place a dipstick into a clean-catch sample of her urine to test for protein c. Reduce her fluid intake to four to five 8-ounce glasses each day d. Do gentle exercises such as hand and feet circles and gently tensing and relaxing arm and leg muscles e. Avoid excessively salty foods f. Maintain strict bed rest in a quiet dimly lighted room with minimal stimuli

20. b, d, and e; magnesium sulfate is administered intravenously in the hospital with severe preeclampsia; a clean catch, midstream urine specimen should be used to assess urine for protein using a dipstick; fluid intake should be 6 to 8 (8 oz.) glasses a day along with roughage to prevent constipation; gentle exercise improves circulation and helps preserve muscle tone and a sense of well-being; modified bed rest with diversional activities is recommended for mild preeclampsia.

21. A woman has just been admitted with a diagnosis of hyperemesis gravidarum. She has been unable to retain any oral intake and as a result has lost weight and is exhibiting signs of dehydration with electrolyte imbalance and acetonuria. The care management of this woman would include which of the following? a. Administering labetalol to control nausea and vomiting b. Assessing the woman's urine for ketones c. Avoiding oral hygiene until the woman is able to tolerate oral fluids d. Providing small frequent meals consisting of bland foods and warm fluids together once the woman begins to respond to treatment

21. b; labetalol is a beta blocker used for hypertension; oral hygiene is important when NPO and after vomiting episodes to maintain the integrity of oral mucosa; taking fluids between, not with, meals reduces nausea, thereby increasing tolerance for oral nutrition.

22. A primigravida at 10 weeks of gestation reports slight vaginal spotting without passage of tissue and mild uterine cramping. When examined, no cervical dilation is noted. The nurse caring for this woman would: a. anticipate that the woman will be sent home and placed on bed rest with instructions to avoid stress and orgasm. b. prepare the woman for a dilation and curettage. c. inform the woman that frequent blood tests will be required to check the level of estrogen. d. tell the woman that the doctor will most likely perform a cerclage to help her maintain her pregnancy.

22. a; the woman is experiencing a threatened abortion; therefore, a conservative approach is attempted first; b reflects management of an inevitable and complete or incomplete abortion; blood tests for HCG and progesterone levels would be done; cerclage or suturing of the cervix is done for recurrent, spontaneous abortion associated with premature dilation of the cervix.

23. A woman is admitted through the emergency room with a medical diagnosis of ruptured ectopic pregnancy. The primary nursing diagnosis at this time would be: a. acute pain related to irritation of the peritoneum with blood. b. risk for infection related to tissue trauma. c. deficient fluid volume related to blood loss associated with rupture of the uterine tube. d. anticipatory grieving related to unexpected pregnancy outcome

23. c; a, b, and d are appropriate nursing diagnoses, but deficient fluid is the most immediate concern, placing the woman's well-being at greatest risk.

24. A woman diagnosed with an ectopic pregnancy is given an intramuscular injection of methotrexate. The nurse would tell the woman which of the following? a. Methotrexate is an analgesic that will relieve the dull abdominal pain she is experiencing. b. She should avoid alcohol until her primary care provider tells her the treatment is complete. c. Follow-up blood tests will be required every other month for 6 months after the injection of the methotrexate. d. She should continue to take her prenatal vitamin and folic acid to enhance healing.

24. b; methotrexate destroys rapidly growing tissue, in this case the fetus and placenta, to avoid rupture of the tube and need for surgery; follow-up with blood tests is needed for 2 to 8 weeks; alcohol and vitamins containing folic acid increase the risk for side effects with this medication or exacerbating the ectopic rupture.

25. A pregnant woman at 32 weeks of gestation comes to the emergency room because she has begun to experience bright red vaginal bleeding. She reports that she is experiencing no pain. The admission nurse suspects: a. abruptio placentae. b. disseminated intravascular coagulation. c. placenta previa. d. preterm labor.

25. c; the clinical manifestations of placenta previa are described; dark red bleeding with pain is characteristic of abruptio placentae; massive bleeding from many sites is associated with DIC; bleeding is not a sign of preterm labor.

26. A pregnant woman, at 38 weeks of gestation diagnosed with marginal placenta previa, has just given birth to a healthy newborn male. The nurse recognizes that the immediate focus for the care of this woman would be: a. preventing hemorrhage. b. relieving pain. c. preventing infection. d. fostering attachment of the woman with her new son.

26. a; hemorrhage is a major potential postpartum complication because the implantation site of theplacenta is in the lower uterine segment, which has a limited capacity to contract after birth; infection is another major complication, but it is not the immediate focus of care; b and d are also important but not to the same degree as hemorrhage, which is life threatening.

The perinatal nurse includes the following when explaining the physiology of artificial rupture of membranes to the student nurse: rupture of membranes causes a release of arachidonic acid, which converts to prostaglandins, substances known to stimulate oxytocin in the pregnant uterus. T/F

ANS: True At certain points in the labor, an amniotomy, or artificial rupture of the membranes, may be successful in increasing uterine contractility.

A multipara, 26 weeks' gestation and accompanied by her husband, has just delivered a fetal demise. Which of the following nursing actions is appropriate at this time? a. Encourage the parents to pray for the baby's soul. b. Advise the parents that it is better for the baby to have died than to have had to live with a defect. c. Encourage the parents to hold the baby. d. Advise the parents to refrain from discussing the baby's death with their other children.

A multipara, 26 weeks' gestation and accompanied by her husband, has just delivered a fetal demise. Which of the following nursing actions is appropriate at this time? a. Encourage the parents to pray for the baby's soul. b. Advise the parents that it is better for the baby to have died than to have had to live with a defect. c. Encourage the parents to hold the baby. d. Advise the parents to refrain from discussing the baby's death with their other children.

Kerry, a 30-year-old G3 TPAL 0110 woman presents to the labor unit triage with complaints of lower abdominal cramping and urinary frequency at 30 weeks gestation. An appropriate nursing action would be to: Select all answers that apply: A) Assess the fetal heart rate B) Obtain a urine for culture and sensitivity C) Assess Kerry's blood pressure and pulse D) Palpate Kerry's abdomen for contractions

A) Assess the fetal heart rate B) Obtain a urine for culture and sensitivity D) Palpate Kerry's abdomen for contractions

The perinatal nurse knows that specific testing is provided to a woman who has had three or more perinatal losses in the first twenty weeks. Appropriate tests may include: Select all answers that apply: A) Cervical cultures B) Sickle cell screening C) Maternal/paternal karyotype D) Thyroid stimulating hormone (TSH) levels

A) Cervical cultures C) Maternal/paternal karyotype D) Thyroid stimulating hormone (TSH) levels

The perinatal nurse describes for the new nurse the various risks associated with prolonged premature preterm rupture of membranes. These risks include: Select all answers that apply: A) Chorioamnionitis B) Abruptio placentae C) Operative birth D) Cord prolapse

A) Chorioamnionitis B) Abruptio placentae D) Cord prolapse

The perinatal nurse describes risk factors for placenta previa to the student nurse. Placenta previa risk factors include: Select all answers that apply: A) Cocaine use B) Tobacco use C) Previous caesarean birth D) Previous use of medroxyprogesterone (Depo-Provera)

A) Cocaine use B) Tobacco use C) Previous caesarean birth

The perinatal nurse provides a hospital tour for couples and families preparing for labor and birth in the future. Teaching is an important component of the tour. Information provided about preterm labor and birth prevention includes: Select all answers that apply: A) Encouraging regular, on-going prenatal care B) Reporting symptoms of urinary frequency and burning to the health care provider C) Coming to the labor triage unit if back pain or cramping persist or become regular D) Lying on the right side, withholding fluids and counting fetal movements if contractions occur every five minutes

A) Encouraging regular, on-going prenatal care B) Reporting symptoms of urinary frequency and burning to the health care provider C) Coming to the labor triage unit if back pain or cramping persist or become regular

The perinatal nurse knows that the survival rate for infants born at or greater than 28-29 gestational weeks is greater than 90%. A) True B) False

A) True

The perinatal nurse observes the placental inspection by the health care provider after birth. This examination may help to determine whether an abruption has occurred prior to, or during labor. A) True B) False

A) True

A post-cesarean section client has been ordered to receive 500 mL of 5% dextrose in water every 4 hours. The drop factor of the macrodrip tubing is 10 gtt/mL. To what drip rate should the nurse regulate the IV? __________ gtt/min

ANS: 21 Feedback: 21 gtt/min The formula for calculating drip rates is: volume multiplied by drop factor = drip rate time in minutes 500 mL = 10 gtt/cc = 21 gtt/min 4 hours = 60 min/hr

33. Spontaneous termination of a pregnancy is considered to be an abortion if: a. The pregnancy is less than 20 weeks. b. The fetus weighs less than 1000 g. c. The products of conception are passed intact. d. No evidence exists of intrauterine infection.

ANS: A An abortion is the termination of pregnancy before the age of viability (20 weeks). The weight of the fetus is not considered because some older fetuses may have a low birth weight. A spontaneous abortion may be complete or incomplete. A spontaneous abortion may be caused by many problems, one being intrauterine infection.

37. The priority nursing intervention when admitting a pregnant woman who has experienced a bleeding episode in late pregnancy is to: a. Assess fetal heart rate (FHR) and maternal vital signs b. Perform a venipuncture for hemoglobin and hematocrit levels c. Place clean disposable pads to collect any drainage d. Monitor uterine contractions

ANS: A Assessment of the FHR and maternal vital signs will assist the nurse in determining the degree of the blood loss and its effect on the mother and fetus. The most important assessment is to check mother/fetal well-being. The blood levels can be obtained later. It is important to assess future bleeding; however, the top priority remains mother/fetal well-being. Monitoring uterine contractions is important but not the top priority.

42. Approximately 10% to 15% of all clinically recognized pregnancies end in miscarriage. Which is the most common cause of spontaneous abortion? a. Chromosomal abnormalities c. Endocrine imbalance b. Infections d. Immunologic factors

ANS: A At least 50% of pregnancy losses result from chromosomal abnormalities that are incompatible with life. Maternal infection may be a cause of early miscarriage. Endocrine imbalances such as hypothyroidism or diabetes are possible causes for early pregnancy loss. Women who have repeated early pregnancy losses appear to have immunologic factors that play a role in spontaneous abortion incidents.

14. Your patient is being induced because of her worsening preeclampsia. She is also receiving magnesium sulfate. It appears that her labor has not become active despite several hours of oxytocin administration. She asks the nurse, "Why is it taking so long?" The most appropriate response by the nurse would be: a. "The magnesium is relaxing your uterus and competing with the oxytocin. It may increase the duration of your labor." b. "I don't know why it is taking so long." c. "The length of labor varies for different women." d. "Your baby is just being stubborn."

ANS: A Because magnesium sulfate is a tocolytic agent, its use may increase the duration of labor. The amount of oxytocin needed to stimulate labor may be more than that needed for the woman who is not receiving magnesium sulfate. "I don't know why it is taking so long" is not an appropriate statement for the nurse to make. Although the length of labor does vary in different women, the most likely reason this woman's labor is protracted is the tocolytic effect of magnesium sulfate. The behavior of the fetus has no bearing on the length of labor.

2. Because pregnant women may need surgery during pregnancy, nurses should be aware that: a. The diagnosis of appendicitis may be difficult because the normal signs and symptoms mimic some normal changes in pregnancy. b. Rupture of the appendix is less likely in pregnant women because of the close monitoring. c. Surgery for intestinal obstructions should be delayed as long as possible because it usually affects the pregnancy. d. When pregnancy takes over, a woman is less likely to have ovarian problems that require invasive responses.

ANS: A Both appendicitis and pregnancy are linked with nausea, vomiting, and increased white blood cell count. Rupture of the appendix is two to three times more likely in pregnant women. Surgery to remove obstructions should be done right away. It usually does not affect the pregnancy. Pregnancy predisposes a woman to ovarian problems.

39. Which order should the nurse expect for a patient admitted with a threatened abortion? a. Bed rest b. Ritodrine IV c. NPO d. Narcotic analgesia every 3 hours, prn

ANS: A Decreasing the woman's activity level may alleviate the bleeding and allow the pregnancy to continue. Ritodrine is not the first drug of choice for tocolytic medications. There is no reason for having the woman placed NPO. At times dehydration may produce contractions, so hydration is important. Narcotic analgesia will not decrease the contractions. It may mask the severity of the contractions.

10. A woman with severe preeclampsia has been receiving magnesium sulfate by intravenous infusion for 8 hours. The nurse assesses the woman and documents the following findings: temperature of 37.1° C, pulse rate of 96 beats/min, respiratory rate of 24 breaths/min, blood pressure (BP) of 155/112 mm Hg, 3+ deep tendon reflexes, and no ankle clonus. The nurse calls the physician, anticipating an order for: a. Hydralazine. c. Diazepam. b. Magnesium sulfate bolus. d. Calcium gluconate.

ANS: A Hydralazine is an antihypertensive commonly used to treat hypertension in severe preeclampsia. Typically it is administered for a systolic BP greater than 160 mm Hg or a diastolic BP greater than 110 mm Hg. An additional bolus of magnesium sulfate may be ordered for increasing signs of central nervous system irritability related to severe preeclampsia (e.g., clonus) or if eclampsia develops. Diazepam sometimes is used to stop or shorten eclamptic seizures. Calcium gluconate is used as the antidote for magnesium sulfate toxicity. The client is not currently displaying any signs or symptoms of magnesium toxicity.

19. In planning care for women with preeclampsia, nurses should be aware that: a. Induction of labor is likely, as near term as possible. b. If at home, the woman should be confined to her bed, even with mild preeclampsia. c. A special diet low in protein and salt should be initiated. d. Vaginal birth is still an option, even in severe cases.

ANS: A Induction of labor is likely, as near term as possible; however, at less than 37 weeks of gestation, immediate delivery may not be in the best interest of the fetus. Strict bed rest is becoming controversial for mild cases; some women in the hospital are even allowed to move around. Diet and fluid recommendations are much the same as for healthy pregnant women, although some authorities have suggested a diet high in protein. Women with severe preeclampsia should expect a cesarean delivery.

16. The nurse caring for pregnant women must be aware that the most common medical complication of pregnancy is: a. Hypertension. c. Hemorrhagic complications. b. Hyperemesis gravidarum. d. Infections.

ANS: A Preeclampsia and eclampsia are two noted deadly forms of hypertension. A large percentage of pregnant women will have nausea and vomiting, but a relatively few have the severe form called hyperemesis gravidarum. Hemorrhagic complications are the second most common medical complication of pregnancy; hypertension is the most common.

5. In caring for the woman with disseminated intravascular coagulation (DIC), what order should the nurse anticipate? a. Administration of blood b. Preparation of the client for invasive hemodynamic monitoring c. Restriction of intravascular fluids d. Administration of steroids

ANS: A Primary medical management in all cases of DIC involves correction of the underlying cause, volume replacement, blood component therapy, optimization of oxygenation and perfusion status, and continued reassessment of laboratory parameters. Central monitoring would not be ordered initially in a client with DIC because this can contribute to more areas of bleeding. Management of DIC would include volume replacement, not volume restriction. Steroids are not indicated for the management of DIC.

15. What nursing diagnosis would be the most appropriate for a woman experiencing severe preeclampsia? a. Risk for injury to the fetus related to uteroplacental insufficiency b. Risk for eclampsia c. Risk for deficient fluid volume related to increased sodium retention secondary to administration of MgSO4 d. Risk for increased cardiac output related to use of antihypertensive drugs

ANS: A Risk for injury to the fetus related to uteroplacental insufficiency is the most appropriate nursing diagnosis for this client scenario. Other diagnoses include Risk to fetus related to preterm birth and abruptio placentae. Eclampsia is a medical, not a nursing, diagnosis. There would be a risk for excess, not deficient, fluid volume related to increased sodium retention. There would be a risk for decreased, not increased, cardiac output related to the use of antihypertensive drugs.

4. In caring for an immediate postpartum client, you note petechiae and oozing from her IV site. You would monitor her closely for the clotting disorder: a. Disseminated intravascular coagulation (DIC) b. Amniotic fluid embolism (AFE) c. Hemorrhage d. HELLP syndrome

ANS: A The diagnosis of DIC is made according to clinical findings and laboratory markers. Physical examination reveals unusual bleeding. Petechiae may appear around a blood pressure cuff on the woman's arm. Excessive bleeding may occur from the site of slight trauma such as venipuncture sites. These symptoms are not associated with AFE, nor is AFE a bleeding disorder. Hemorrhage occurs for a variety of reasons in the postpartum client. These symptoms are associated with DIC. Hemorrhage would be a finding associated with DIC and is not a clotting disorder in and of itself. HELLP is not a clotting disorder, but it may contribute to the clotting disorder DIC.

45. The reported incidence of ectopic pregnancy in the United States has risen steadily over the past 2 decades. Causes include the increase in STDs accompanied by tubal infection and damage. The popularity of contraceptive devices such as the IUD has also increased the risk for ectopic pregnancy. The nurse who suspects that a patient has early signs of ectopic pregnancy should be observing her for symptoms such as (Select all that apply): a. Pelvic pain b. Abdominal pain c. Unanticipated heavy bleeding d. Vaginal spotting or light bleeding e. Missed period

ANS: A, B, D, E A missed period or spotting can easily be mistaken by the patient as early signs of pregnancy. More subtle signs depend on exactly where the implantation occurs. The nurse must be thorough in her assessment because pain is not a normal symptom of early pregnancy. As the fallopian tube tears open and the embryo is expelled, the patient often exhibits severe pain accompanied by intraabdominal hemorrhage. This may progress to hypovolemic shock with minimal or even no external bleeding. In about half of women, shoulder and neck pain results from irritation of the diaphragm from the hemorrhage.

44. A client who has undergone a dilation and curettage for early pregnancy loss is likely to be discharged the same day. The nurse must ensure that vital signs are stable, bleeding has been controlled, and the woman has adequately recovered from the administration of anesthesia. To promote an optimal recovery, discharge teaching should include (Select all that apply): a. Iron supplementation. b. Resumption of intercourse at 6 weeks following the procedure. c. Referral to a support group if necessary. d. Expectation of heavy bleeding for at least 2 weeks. e. Emphasizing the need for rest.

ANS: A, C, E The woman should be advised to consume a diet high in iron and protein. For many women iron supplementation also is necessary. Acknowledge that the client has experienced a loss, albeit early. She can be taught to expect mood swings and possibly depression. Referral to a support group, clergy, or professional counseling may be necessary. Discharge teaching should emphasize the need for rest. Nothing should be placed in the vagina for 2 weeks after the procedure. This includes tampons and vaginal intercourse. The purpose of this recommendation is to prevent infection. Should infection occur, antibiotics may be prescribed. The client should expect a scant, dark discharge for 1 to 2 weeks. Should heavy, profuse, or bright bleeding occur, she should be instructed to contact her provider.

12. The patient that you are caring for has severe preeclampsia and is receiving a magnesium sulfate infusion. You become concerned after assessment when the woman exhibits: a. A sleepy, sedated affect. c. Deep tendon reflexes of 2. b. A respiratory rate of 10 breaths/min. d. Absent ankle clonus.

ANS: B A respiratory rate of 10 breaths/min indicates that the client is experiencing respiratory depression from magnesium toxicity. Because magnesium sulfate is a central nervous system depressant, the client will most likely become sedated when the infusion is initiated. Deep tendon reflexes of 2 and absent ankle clonus are normal findings.

3. What laboratory marker is indicative of disseminated intravascular coagulation (DIC)? a. Bleeding time of 10 minutes c. Thrombocytopenia b. Presence of fibrin split products d. Hyperfibrinogenemia

ANS: B Degradation of fibrin leads to the accumulation of fibrin split products in the blood. Bleeding time in DIC is normal. Low platelets may occur with but are not indicative of DIC because they may result from other coagulopathies. Hypofibrinogenemia would occur with DIC.

38. A patient with pregnancy-induced hypertension is admitted complaining of pounding headache, visual changes, and epigastric pain. Nursing care is based on the knowledge that these signs are an indication of: a. Anxiety due to hospitalization. b. Worsening disease and impending convulsion. c. Effects of magnesium sulfate. d. Gastrointestinal upset.

ANS: B Headache and visual disturbances are caused by increased cerebral edema. Epigastric pain indicates distention of the hepatic capsules and often warns that a convulsion is imminent. These are danger signs showing increased cerebral edema and impending convulsion and should be treated immediately. The patient has not been started on magnesium sulfate treatment yet. Also, these are not anticipated effects of the medication.

32. Which maternal condition always necessitates delivery by cesarean section? a. Partial abruptio placentae c. Ectopic pregnancy b. Total placenta previa d. Eclampsia

ANS: B In total placenta previa, the placenta completely covers the cervical os. The fetus would die if a vaginal delivery occurred. If the mother has stable vital signs and the fetus is alive, a vaginal delivery can be attempted in cases of partial abruptio placentae. If the fetus has died, a vaginal delivery is preferred. The most common ectopic pregnancy is a tubal pregnancy, which is usually detected and treated in the first trimester. Labor can be safely induced if the eclampsia is under control.

43. The nurse caring for a woman hospitalized for hyperemesis gravidarum should expect that initial treatment to involve: a. Corticosteroids to reduce inflammation. b. IV therapy to correct fluid and electrolyte imbalances. c. An antiemetic, such as pyridoxine, to control nausea and vomiting. d. Enteral nutrition to correct nutritional deficits.

ANS: B Initially, the woman who is unable to keep down clear liquids by mouth requires IV therapy for correction of fluid and electrolyte imbalances. Corticosteroids have been used successfully to treat refractory hyperemesis gravidarum; however, they are not the expected initial treatment for this disorder. Pyridoxine is vitamin B6, not an antiemetic. Promethazine, a common antiemetic, may be prescribed. In severe cases of hyperemesis gravidarum, enteral nutrition via a feeding tube may be necessary to correct maternal nutritional deprivation. This is not an initial treatment for this patient.

34. An abortion in which the fetus dies but is retained within the uterus is called a(n): a. Inevitable abortion c. Incomplete abortion b. Missed abortion d. Threatened abortion

ANS: B Missed abortion refers to retention of a dead fetus in the uterus. An inevitable abortion means that the cervix is dilating with the contractions. An incomplete abortion means that not all of the products of conception were expelled. With a threatened abortion the woman has cramping and bleeding but not cervical dilation.

24. The most prevalent clinical manifestation of abruptio placentae (as opposed to placenta previa) is: a. Bleeding. c. Uterine activity. b. Intense abdominal pain. d. Cramping.

ANS: B Pain is absent with placenta previa and may be agonizing with abruptio placentae. Bleeding may be present in varying degrees for both placental conditions. Uterine activity and cramping may be present with both placental conditions.

26. A 26-year-old pregnant woman, gravida 2, para 1-0-0-1 is 28 weeks pregnant when she experiences bright red, painless vaginal bleeding. On her arrival at the hospital, what would be an expected diagnostic procedure? a. Amniocentesis for fetal lung maturity b. Ultrasound for placental location c. Contraction stress test (CST) d. Internal fetal monitoring

ANS: B The presence of painless bleeding should always alert the health care team to the possibility of placenta previa. This can be confirmed through ultrasonography. Amniocentesis would not be performed on a woman who is experiencing bleeding. In the event of an imminent delivery, the fetus would be presumed to have immature lungs at this gestational age, and the mother would be given corticosteroids to aid in fetal lung maturity. A CST would not be performed at a preterm gestational age. Furthermore, bleeding would be a contraindication to this test. Internal fetal monitoring would be contraindicated in the presence of bleeding.

23. The perinatal nurse is giving discharge instructions to a woman after suction curettage secondary to a hydatidiform mole. The woman asks why she must take oral contraceptives for the next 12 months. The best response from the nurse would be: a. "If you get pregnant within 1 year, the chance of a successful pregnancy is very small. Therefore, if you desire a future pregnancy, it would be better for you to use the most reliable method of contraception available." b. "The major risk to you after a molar pregnancy is a type of cancer that can be diagnosed only by measuring the same hormone that your body produces during pregnancy. If you were to get pregnant, it would make the diagnosis of this cancer more difficult." c. "If you can avoid a pregnancy for the next year, the chance of developing a second molar pregnancy is rare. Therefore, to improve your chance of a successful pregnancy, it is better not to get pregnant at this time." d. "Oral contraceptives are the only form of birth control that will prevent a recurrence of a molar pregnancy."

ANS: B This is an accurate statement. β-Human chorionic gonadotropin (hCG) levels will be drawn for 1 year to ensure that the mole is completely gone. There is an increased chance of developing choriocarcinoma after the development of a hydatidiform mole. The goal is to achieve a "zero" hCG level. If the woman were to become pregnant, it could obscure the presence of the potentially carcinogenic cells. Women should be instructed to use birth control for 1 year after treatment for a hydatidiform mole. The rationale for avoiding pregnancy for 1 year is to ensure that carcinogenic cells are not present. Any contraceptive method except an intrauterine device is acceptable.

36. What condition indicates concealed hemorrhage when the patient experiences an abruptio placentae? a. Decrease in abdominal pain c. Hard, boardlike abdomen b. Bradycardia d. Decrease in fundal height

ANS: C Concealed hemorrhage occurs when the edges of the placenta do not separate. The formation of a hematoma behind the placenta and subsequent infiltration of the blood into the uterine muscle results in a very firm, boardlike abdomen. Abdominal pain may increase. The patient will have shock symptoms that include tachycardia. As bleeding occurs, the fundal height will increase.

27. A laboring woman with no known risk factors suddenly experiences spontaneous rupture of membranes (ROM). The fluid consists of bright red blood. Her contractions are consistent with her current stage of labor. There is no change in uterine resting tone. The fetal heart rate begins to decline rapidly after the ROM. The nurse should suspect the possibility of: a. Placenta previa. b. Vasa previa. c. Severe abruptio placentae. d. Disseminated intravascular coagulation (DIC).

ANS: B Vasa previa is the result of a velamentous insertion of the umbilical cord. The umbilical vessels are not surrounded by Wharton jelly and have no supportive tissue. They are at risk for laceration at any time, but laceration occurs most frequently during ROM. The sudden appearance of bright red blood at the time of ROM and a sudden change in the fetal heart rate without other known risk factors should immediately alert the nurse to the possibility of vasa previa. The presence of placenta previa most likely would be ascertained before labor and would be considered a risk factor for this pregnancy. In addition, if the woman had a placenta previa, it is unlikely that she would be allowed to pursue labor and a vaginal birth. With the presence of severe abruptio placentae, the uterine tonicity would typically be tetanus (i.e., a boardlike uterus). DIC is a pathologic form of diffuse clotting that consumes large amounts of clotting factors and causes widespread external bleeding, internal bleeding, or both. DIC is always a secondary diagnosis, often associated with obstetric risk factors such as HELLP syndrome. This woman did not have any prior risk factors.

. Women with hyperemesis gravidarum: a. Are a majority, because 80% of all pregnant women suffer from it at some time. b. Have vomiting severe and persistent enough to cause weight loss, dehydration, and electrolyte imbalance. c. Need intravenous (IV) fluid and nutrition for most of their pregnancy. d. Often inspire similar, milder symptoms in their male partners and mothers.

ANS: B Women with hyperemesis gravidarum have severe vomiting; however, treatment for several days sets things right in most cases. Although 80% of pregnant women experience nausea and vomiting, fewer than 1% (0.5%) proceed to this severe level. IV administration may be used at first to restore fluid levels, but it is seldom needed for very long. Women suffering from this condition want sympathy because some authorities believe that difficult relationships with mothers and/or partners may be the cause.

22. A woman presents to the emergency department with complaints of bleeding and cramping. The initial nursing history is significant for a last menstrual period 6 weeks ago. On sterile speculum examination, the primary care provider finds that the cervix is closed. The anticipated plan of care for this woman would be based on a probable diagnosis of which type of spontaneous abortion? a. Incomplete c. Threatened b. Inevitable d. Septic

ANS: C A woman with a threatened abortion presents with spotting, mild cramps, and no cervical dilation. A woman with an incomplete abortion would present with heavy bleeding, mild to severe cramping, and cervical dilation. An inevitable abortion manifests with the same symptoms as an incomplete abortion: heavy bleeding, mild to severe cramping, and cervical dilation. A woman with a septic abortion presents with malodorous bleeding and typically a dilated cervix.

28. A woman arrives for evaluation of her symptoms, which include a missed period, adnexal fullness, tenderness, and dark red vaginal bleeding. On examination the nurse notices an ecchymotic blueness around the woman's umbilicus and recognizes this assessment finding as: a. Normal integumentary changes associated with pregnancy. b. Turner's sign associated with appendicitis. c. Cullen's sign associated with a ruptured ectopic pregnancy. d. Chadwick's sign associated with early pregnancy.

ANS: C Cullen's sign, the blue ecchymosis seen in the umbilical area, indicates hematoperitoneum associated with an undiagnosed ruptured intraabdominal ectopic pregnancy. Linea nigra on the abdomen is the normal integumentary change associated with pregnancy. It manifests as a brown, pigmented, vertical line on the lower abdomen. Turner's sign is ecchymosis in the flank area, often associated with pancreatitis. Chadwick's sign is the blue-purple color of the cervix that may be seen during or around the eighth week of pregnancy.

7. The labor of a pregnant woman with preeclampsia is going to be induced. Before initiating the Pitocin infusion, the nurse reviews the woman's latest laboratory test findings, which reveal a platelet count of 90,000, an elevated aspartate transaminase (AST) level, and a falling hematocrit. The nurse notifies the physician because the laboratory results are indicative of: a. Eclampsia. b. Disseminated intravascular coagulation (DIC). c. HELLP syndrome. d. Idiopathic thrombocytopenia.

ANS: C HELLP syndrome is a laboratory diagnosis for a variant of severe preeclampsia that involves hepatic dysfunction characterized by hemolysis (H), elevated liver enzymes (EL), and low platelets (LP). Eclampsia is determined by the presence of seizures. DIC is a potential complication associated with HELLP syndrome. Idiopathic thrombocytopenia is the presence of low platelets of unknown cause and is not associated with preeclampsia.

31. In providing nutritional counseling for the pregnant woman experiencing cholecystitis, the nurse would: a. Assess the woman's dietary history for adequate calories and proteins. b. Instruct the woman that the bulk of calories should come from proteins. c. Instruct the woman to eat a low-fat diet and avoid fried foods. d. Instruct the woman to eat a low-cholesterol, low-salt diet.

ANS: C Instructing the woman to eat a low-fat diet and avoid fried foods is appropriate nutritional counseling for this client. Caloric and protein intake do not predispose a woman to the development of cholecystitis. The woman should be instructed to limit protein intake and choose foods that are high in carbohydrates. A low-cholesterol diet may be the result of limiting fats. However, a low-salt diet is not indicated.

20. Magnesium sulfate is given to women with preeclampsia and eclampsia to: a. Improve patellar reflexes and increase respiratory efficiency. b. Shorten the duration of labor. c. Prevent and treat convulsions. d. Prevent a boggy uterus and lessen lochial flow.

ANS: C Magnesium sulfate is the drug of choice to prevent convulsions, although it can generate other problems. Loss of patellar reflexes and respiratory depression are signs of magnesium toxicity. Magnesium sulfate can increase the duration of labor. Women are at risk for a boggy uterus and heavy lochial flow as a result of magnesium sulfate therapy.

25. Methotrexate is recommended as part of the treatment plan for which obstetric complication? a. Complete hydatidiform mole c. Unruptured ectopic pregnancy b. Missed abortion d. Abruptio placentae

ANS: C Methotrexate is an effective, nonsurgical treatment option for a hemodynamically stable woman whose ectopic pregnancy is unruptured and less than 4 cm in diameter. Methotrexate is not indicated or recommended as a treatment option for complete hydatidiform mole, missed abortion, and abruptio placentae.

6. A primigravida is being monitored in her prenatal clinic for preeclampsia. What finding should concern her nurse? a. Blood pressure (BP) increase to 138/86 mm Hg b. Weight gain of 0.5 kg during the past 2 weeks c. A dipstick value of 3+ for protein in her urine d. Pitting pedal edema at the end of the day

ANS: C Proteinuria is defined as a concentration of 1+ or greater via dipstick measurement. A dipstick value of 3+ should alert the nurse that additional testing or assessment should be made. Generally, hypertension is defined as a BP of 140/90 or an increase in systolic pressure of 30 mm Hg or in diastolic pressure of 15 mm Hg. Preeclampsia may be manifested as a rapid weight gain of more than 2 kg in 1 week. Edema occurs in many normal pregnancies and in women with preeclampsia. Therefore, the presence of edema is no longer considered diagnostic of preeclampsia.

30. Which condition would not be classified as a bleeding disorder in late pregnancy? a. Placenta previa. c. Spontaneous abortion. b. Abruptio placentae. d. Cord insertion.

ANS: C Spontaneous abortion is another name for miscarriage; by definition it occurs early in pregnancy. Placenta previa is a cause of bleeding disorders in later pregnancy. Abruptio placentae is a cause of bleeding disorders in later pregnancy. Cord insertion is a cause of bleeding disorders in later pregnancy.

35. A placenta previa in which the placental edge just reaches the internal os is more commonly known as: a. Total c. Complete b. Partial d. Marginal

ANS: D A placenta previa that does not cover any part of the cervix is termed marginal. With a total placenta previa, the placenta completely covers the os. When the patient experiences a partial placenta previa, the lower border of the placenta is within 3 cm of the internal cervical os but does not completely cover the os. A complete placenta previa is termed total. The placenta completely covers the internal cervical os.

17. Nurses should be aware that HELLP syndrome: a. Is a mild form of preeclampsia. b. Can be diagnosed by a nurse alert to its symptoms. c. Is characterized by hemolysis, elevated liver enzymes, and low platelets. d. Is associated with preterm labor but not perinatal mortality.

ANS: C The acronym HELLP stands for hemolysis (H), elevated liver enzymes (EL), and low platelets (LP). HELLP syndrome is a variant of severe preeclampsia. HELLP syndrome is difficult to identify because the symptoms often are not obvious. It must be diagnosed in the laboratory. Preterm labor is greatly increased, and so is perinatal mortality.

9. A pregnant woman has been receiving a magnesium sulfate infusion for treatment of severe preeclampsia for 24 hours. On assessment the nurse finds the following vital signs: temperature of 37.3° C, pulse rate of 88 beats/min, respiratory rate of 10 breaths/min, blood pressure (BP) of 148/90 mm Hg, absent deep tendon reflexes, and no ankle clonus. The client complains, "I'm so thirsty and warm." The nurse: a. Calls for a stat magnesium sulfate level. b. Administers oxygen. c. Discontinues the magnesium sulfate infusion. d. Prepares to administer hydralazine.

ANS: C The client is displaying clinical signs and symptoms of magnesium toxicity. Magnesium should be discontinued immediately. In addition, calcium gluconate, the antidote for magnesium, may be administered. Hydralazine is an antihypertensive commonly used to treat hypertension in severe preeclampsia. Typically it is administered for a systolic BP greater than 160 mm Hg or a diastolic BP greater than 110 mm Hg.

40. What finding on a prenatal visit at 10 weeks could suggest a hydatidiform mole? a. Complaint of frequent mild nausea b. Blood pressure of 120/80 mm Hg c. Fundal height measurement of 18 cm d. History of bright red spotting for 1 day, weeks ago

ANS: C The uterus in a hydatidiform molar pregnancy is often larger than would be expected on the basis of the duration of the pregnancy. Nausea increases in a molar pregnancy because of the increased production of hCG. A woman with a molar pregnancy may have early-onset pregnancy-induced hypertension. In the patient's history, bleeding is normally described as brownish.

21. Preeclampsia is a unique disease process related only to human pregnancy. The exact cause of this condition continues to elude researchers. The American College of Obstetricians and Gynecologists has developed a comprehensive list of risk factors associated with the development of preeclampsia. Which client exhibits the greatest number of these risk factors? a. A 30-year-old obese Caucasian with her third pregnancy b. A 41-year-old Caucasian primigravida c. An African-American client who is 19 years old and pregnant with twins d. A 25-year-old Asian-American whose pregnancy is the result of donor insemination

ANS: C Three risk factors are present for this woman. She is of African-American ethnicity, is at the young end of the age distribution, and has a multiple pregnancy. In planning care for this client the nurse must monitor blood pressure frequently and teach the woman regarding early warning signs. The 30-year-old client only has one known risk factor, obesity. Age distribution appears to be U-shaped, with women less than 20 years and more than 40 years being at greatest risk. Preeclampsia continues to be seen more frequently in primigravidas; this client is a multigravida woman. Two risk factors are present for the 41-year-old client. Her age and status as a primigravida put her at increased risk for preeclampsia. Caucasian women are at a lower risk than African-American women. The Asian-American client exhibits only one risk factor. Pregnancies that result from donor insemination, oocyte donation, and embryo donation are at an increased risk of developing preeclampsia.

__________ is contraindicated with shoulder dystocia.

ANS: Fundal pressureFundal pressure is contraindicated with shoulder dystocia because it may further impact the shoulder and increases risk of fetal injury.

13. Your patient has been receiving magnesium sulfate for 20 hours for treatment of preeclampsia. She just delivered a viable infant girl 30 minutes ago. What uterine findings would you expect to observe/assess in this client? a. Absence of uterine bleeding in the postpartum period b. A fundus firm below the level of the umbilicus c. Scant lochia flow d. A boggy uterus with heavy lochia flow

ANS: D Because of the tocolytic effects of magnesium sulfate, this patient most likely would have a boggy uterus with increased amounts of bleeding and a heavy lochia flow in the postpartum period.

29. As related to the care of the patient with miscarriage, nurses should be aware that: a. It is a natural pregnancy loss before labor begins. b. It occurs in fewer than 5% of all clinically recognized pregnancies. c. It often can be attributed to careless maternal behavior such as poor nutrition or excessive exercise. d. If it occurs before the twelfth week of pregnancy, it may manifest only as moderate discomfort and blood loss.

ANS: D Before the sixth week the only evidence may be a heavy menstrual flow. After the twelfth week more severe pain, similar to that of labor, is likely. Miscarriage is a natural pregnancy loss, but by definition it occurs before 20 weeks of gestation, before the fetus is viable. Miscarriages occur in approximately 10% to 15% of all clinically recognized pregnancies. Miscarriage can be caused by a number of disorders or illnesses outside of the mother's control or knowledge.

18. Nurses should be aware that chronic hypertension: a. Is defined as hypertension that begins during pregnancy and lasts for the duration of pregnancy. b. Is considered severe when the systolic blood pressure (BP) is greater than 140 mm Hg or the diastolic BP is greater than 90 mm Hg. c. Is general hypertension plus proteinuria. d. Can occur independently of or simultaneously with gestational hypertension.

ANS: D Hypertension is present before pregnancy or diagnosed before 20 weeks of gestation and persists longer than 6 weeks postpartum. The range for hypertension is systolic BP greater than 140 mm Hg or diastolic BP greater than 90 mm Hg. It becomes severe with a diastolic BP of 110 mm Hg or higher. Proteinuria is an excessive concentration of protein in the urine. It is a complication of hypertension, not a defining characteristic.

8. A woman with preeclampsia has a seizure. The nurse's primary duty during the seizure is to: a. Insert an oral airway. b. Suction the mouth to prevent aspiration. c. Administer oxygen by mask. d. Stay with the client and call for help.

ANS: D If a client becomes eclamptic, the nurse should stay her and call for help. Insertion of an oral airway during seizure activity is no longer the standard of care. The nurse should attempt to keep the airway patent by turning the client's head to the side to prevent aspiration. Once the seizure has ended, it may be necessary to suction the client's mouth. Oxygen would be administered after the convulsion has ended.

41. A 32-year-old primigravida is admitted with a diagnosis of ectopic pregnancy. Nursing care is based on the knowledge that: a. Bed rest and analgesics are the recommended treatment. b. She will be unable to conceive in the future. c. A D&C will be performed to remove the products of conception. d. Hemorrhage is the major concern.

ANS: D Severe bleeding occurs if the fallopian tube ruptures. The recommended treatment is to remove the pregnancy before rupture in order to prevent hemorrhaging. If the tube must be removed, the woman's fertility will decrease; however, she will not be infertile. D&C is performed on the inside of the uterine cavity. The ectopic pregnancy is located within the tubes.

11. A woman at 39 weeks of gestation with a history of preeclampsia is admitted to the labor and birth unit. She suddenly experiences increased contraction frequency of every 1 to 2 minutes; dark red vaginal bleeding; and a tense, painful abdomen. The nurse suspects the onset of: a. Eclamptic seizure. c. Placenta previa. b. Rupture of the uterus. d. Placental abruption.

ANS: D Uterine tenderness in the presence of increasing tone may be the earliest finding of premature separation of the placenta (abruptio placentae or placental abruption). Women with hypertension are at increased risk for an abruption. Eclamptic seizures are evidenced by the presence of generalized tonic-clonic convulsions. Uterine rupture manifests as hypotonic uterine activity, signs of hypovolemia, and in many cases the absence of pain. Placenta previa manifests with bright red, painless vaginal bleeding.

The postpartum nurse is caring for a couple who experienced an unplanned emergency cesarean birth. The nurse observes the following behaviors: Parents are gently touching their newborn. Mother is softly singing to her baby. Father is gazing into his baby's eyes. Based on this data, the correct nursing diagnosis is altered parent-infant bonding related to emergency cesarean birth. Cesarean birth can place the parents at risk for bonding, but based on the observed interaction with their newborn, the parents display positive signs of bonding. T/F

ANS: False

Bonding is bidirectional from parent to infant and infant to parent. T/F

ANS: False Bonding is unidirectional from parent to infant. Attachment is bidirectional.

During an emergency cesarean birth the "time-out" procedure may be omitted based on the obstetrical emergency. T/F

ANS: False Joint commission guidelines for patient safety necessitate there always be a time-out to prevent wrong patient, wrong site, wrong procedure, and medical errors.

True or false: 39. It is critical for the perinatal nurse to learn, as part of the facility's policies and procedures, to immediately perform a vaginal examination on a woman who presents with vaginal bleeding after 24 weeks' gestation.

ANS: False Placenta previa should be suspected in all patients who present with bleeding after 24 completed weeks of gestation. Because of the risk of placental perforation, vaginal examinations are not performed.

The perinatal nurse explains to the student nurse that the most frequent fetal risk associated with the use of forceps is cord compression. T/F

ANS: False The most frequent fetal risk associated with the use of forceps is superficial scalp or facial marks that will resolve quickly.

True or false: 37. Immediately postpartum, the insulin needs in diabetic women increase dramatically.

ANS: FalseThere is a significant decrease in the need for insulin immediately after delivery related to the loss of antagonistic placental hormones and suppression of the anterior pituitary growth hormone.

True or false: 41. A patient with hypertension who is receiving intravenous magnesium sulfate therapy has requested an epidural anesthetic. The perinatal nurse should first review the patient's complete blood count results for evidence of a decreased platelet count.

ANS: True Baseline information, including complete blood count (CBC), clotting studies, serum electrolytes, and renal function tests, is used to alert the care providers to changes in the patient's condition as additional laboratory tests are obtained.

True or false: 42. The perinatal nurse knows that the laboring diabetic patient's blood glucose level should always be less than 120 mg/dL.

ANS: True Blood glucose levels are assessed every hour, and fluid/insulin adjustments are made as needed to maintain maternal blood glucose levels between 80 and 120 mg/dL.

Eye movements are an example of newborn/infant style of communication. T/F

ANS: True Crying, cooing, facial expressions, eye movements, cuddling, and arm and leg movements are all examples of newborn/infant style of communication.

The perinatal nurse describes asynclitism to students as a presentation that occurs when the fetal head is turned toward the maternal sacrum or symphysis at an oblique angle. T/F

ANS: True Face and brow presentations are examples of asynclitism (the fetal head is presenting at a different angle than expected). Face and brow presentations hyperextend the neck and increase the overall circumference of the presenting part. These presentations are uncommon and are usually associated with fetal anomalies.

True or false: 38. The perinatal nurse observes the placental inspection by the health-care provider after birth. This examination may help to determine whether an abruption has occurred prior to or during labor.

ANS: True Fifty percent of abruptions occur before labor and after the 30th week, 15% occur during labor, and 30% are identified only upon inspection of the placenta after delivery.

Metritis is an infection that usually starts at the placental site. T/F

ANS: True Metritis is an infection of the endometrium that usually starts at the placental site and spreads to encompass the entire endometrium.

Abruptio placenta is a risk factor for amniotic fluid embolism. T/F

ANS: True Risk factors for amniotic fluid embolism include induction of labor, maternal age over 35, operative delivery, placenta previa, abruptio placenta, polyhydramnios, eclampsia, and cervical or uterine lacerations.

True or false: 40. The perinatal nurse knows that the survival rate for infants born at or greater than 28 to 29 gestational weeks is greater than 90%.

ANS: True With appropriate medical care, neonatal survival dramatically improves as the gestational age increases, with over 50% of neonates surviving at 25 weeks' gestation, and over 90% surviving at 28 to 29 weeks of gestation.

15. The perinatal nurse is assisting the student nurse with completion of documentation. The laboring woman has just given birth to a 2700 gram infant at 36 weeks' gestation. The most appropriate term for this is: a. Preterm birth b. Term birth c. Small for gestational age infant d. Large for gestational age infant

ANS: a a. A preterm infant is an infant with gestational age of fewer than 36 completed weeks. b. Term births are infants born between 37 and 40 weeks. c. SAG infants at 36 weeks weigh less than 2000 grams. d. LAG infants at 36 weeks weigh over 3400 grams.

10. A 42-week gestation neonate is admitted to the NICU (neonatal intensive care unit). This neonate is at risk for which complication? a. Meconium aspiration syndrome b. Failure to thrive c. Necrotizing enterocolitis d. Intraventricular hemorrhage

ANS: a a. Although there is nothing in the scenario that states that the amniotic fluid is green tinged, post-term babies are high risk for meconium aspiration syndrome. b. Post-term babies often gain weight very quickly. c. Preterm, not post-term, babies are high risk for necrotizing enterocolitis. d. Preterm, not post-term, babies are high risk for intraventricular hemorrhages.

13. The laboratory reported that the L/S ratio (lecithin/sphingomyelin) results from an amniocentesis of a gravid client with preeclampsia are 2:1. The nurse interprets the result as which of the following? a. The baby's lung fields are mature. b. The mother is high risk for hemorrhage. c. The baby's kidneys are functioning poorly. d. The mother is high risk for eclampsia

ANS: a a. An L/S ratio of 2:1 usually indicates that the fetal lungs are mature. b. L/S ratios are unrelated to maternal blood loss. c. L/S ratios are unrelated to fetal renal function. d. L/S ratios are unrelated to maternal risk for becoming eclamptic.

12. A baby boy was just born to a mother who had positive vaginal cultures for group B streptococci. The mother was admitted to the labor room 30 minutes before the birth. For which of the following should the nursery nurse closely observe this baby? a. Grunting b. Acrocyanosis c. Pseudostrabismus d. Hydrocele

ANS: a a. This infant is high risk for respiratory distress. The nurse should observe this baby carefully for grunting. b. Acrocyanosis is a normal finding. c. Pseudostrabismus is a normal finding. d. Hydrocele should be reported to the neonatologist. It is not, however, an emergent problem, and it is not related to group B streptococci colonization in the mother.

Approximately 8 hours ago, Juanita, a 32-year-old G1 P0, gave birth after 2 ½ hours of pushing. She required an episiotomy and an assisted birth (forceps) due to the weight and size of her baby (9 lb. 9 oz.). The perinatal nurse is performing an assessment of Juanita's perineal area. A slight bulge is palpated and the presence of ecchymoses to the right of the episiotomy is noted. The area feels "full" and is approximately 4 cm in diameter. Juanita describes this area as "very tender." The most likely cause of these signs and symptoms is: a. Hematoma formation b. Sepsis in the episiotomy site c. Inadequate repair of the episiotomy d. Postpartum hemorrhage

ANS: a A hematoma is a localized collection of blood in connective or soft tissue under the skin that follows injury of or laceration to a blood vessel without injury to the overlying tissue. The most common sign or symptom of a hematoma is unremitting pain and pressure. Upon examination of the perineal or vulvar areas, the nurse may notice discoloration and bulging of the tissue at the hematoma site. If touched, the patient complains of severe tenderness, and the clinician generally describes the tissue as "full."

The best time to give prophylactic antibiotics to the women undergoing cesarean section is: a. One hour before the surgery b. Two hours before the surgery c. Not indicated unless she has an active infection d. At the time the cord is clamped

ANS: a Administration of narrow-spectrum prophylactic antibiotics should occur within 60 minutes prior to the skin incision.

A pregnant woman who has a history of cesarean births is requesting to have a vaginal birth after cesarean (VBAC). In which of the following situations should the nurse advise the patient that her request may be declined? a. Transverse fetal lie b. Flexed fetal attitude c. Previous low flap uterine incision d. Positive vaginal candidiasis

ANS: a Feedback a. A baby in the transverse lie is lying sideways in the uterus. This lie is incompatible physiologically with a vaginal delivery. b. A baby in the flexed fetal attitude is in a physiologic position for a vaginal delivery. c. A previous low flap uterine incision is not incompatible physiologically with a vaginal delivery. d. A positive vaginal Candidiasis culture is not an indication for cesarean birth.

4. Which of the following assessments would indicate instability in the client hospitalized for placenta previa? a. BP <90/60 mm/Hg, Pulse <60 BPM or >120 BPM b. FHR moderate variability without accelerations c. Dark brown vaginal discharge when voiding d. Oral temperature of 99.9°F

ANS: a Feedback a. A decrease in BP accompanied by bradycardia or tachycardia is an indication of hypovolemic shock. b. FHR with moderate variability can be absent of accelerations during fetal sleep cycles or after maternal sedation. c. Bright red vaginal bleeding is an indication of current bleeding. d. Oral temperature may fluctuate based on the client's hydration status. It should be reassessed. Cause for concern is a temperature of 100.4°F or more.

Four women are close to delivery on the labor and delivery unit. The nurse knows to be vigilant to the signs of neonatal respiratory distress in which delivery? a. 42-week-gestation pregnancy complicated by intrauterine growth restriction b. 41-week-gestation pregnancy with biophysical profile score of 10 that morning c. 40-week-gestation pregnancy with estimated fetal weight of 3200 grams d. 39-week-gestation pregnancy complicated by maternal cholecystitis

ANS: a Feedback a. A post-term baby with intrauterine growth restriction (IUGR) is high risk for meconium aspiration syndrome, cold stress syndrome, hypoglycemia, and acidosis. In each case, the baby may exhibit signs of respiratory distress. b. A biophysical profile (BPP) of 10 is a normal finding. c. The normal birth weight is between 2500 and 4000 grams. d. Maternal gallbladder disease does not place the baby in danger of developing respiratory distress.

The perinatal nurse is assisting the student nurse with completion of documentation. The laboring woman has just given birth to a 2700 gram infant at 36 weeks' gestation. The most appropriate term for this is: a. Preterm birth b. Term birth c. Small for gestational age infant d. Large for gestational age infant

ANS: a Feedback a. A preterm infant is an infant with gestational age of fewer than 36 completed weeks. b. Term births are infants born between 37 and 40 weeks. c. SAG infants at 36 weeks weigh less than 2000 grams. d. LAG infants at 36 weeks weigh over 3400 grams.

The clinic nurse meets with Rebecca, a 30-year-old woman who is experiencing her first pregnancy. Rebecca's quadruple marker screen result is positive at 17 weeks' gestation. The nurse explains that Rebecca needs a referral to: a. A genetics counselor/specialist b. An obstetrician c. A gynecologist d. A social worker

ANS: a Feedback a. All women should be offered screening with maternal serum markers. The Triple Marker screen and the Quadruple Marker screen test for the presence of alpha-fetoprotein (AFP), estradiol, human chorionic gonadotropin (hCG), and other markers. These tests screen for potential neural tube defects, Down syndrome, and Trisomy 18. If the screen is positive, the woman should be referred to a genetics specialist for counseling, and further testing, such as chorionic villus sampling (CVS) or amniocentesis, should be performed. b. If genetic screening is positive, the woman should be referred to a genetics specialist for counseling, and further testing, such as chorionic villus sampling (CVS) or amniocentesis, should be performed. c. If genetic screening is positive, the woman should be referred to a genetics specialist for counseling, and further testing, such as chorionic villus sampling (CVS) or amniocentesis, should be performed. d. If genetic screening is positive, the woman should be referred to a genetics specialist for counseling, and further testing, such as chorionic villus sampling (CVS) or amniocentesis, should be performed.

A 42-week gestation neonate is admitted to the NICU (neonatal intensive care unit). This neonate is at risk for which complication? a. Meconium aspiration syndrome b. Failure to thrive c. Necrotizing enterocolitis d. Intraventricular hemorrhage

ANS: a Feedback a. Although there is nothing in the scenario that states that the amniotic fluid is green tinged, post-term babies are high risk for meconium aspiration syndrome. b. Post-term babies often gain weight very quickly. c. Preterm, not post-term, babies are high risk for necrotizing enterocolitis. d. Preterm, not post-term, babies are high risk for intraventricular hemorrhages.

A laboratory report indicates the L/S ratio (lecithin/sphingomyelin) results from an amniocentesis of a gravid patient with preeclampsia are 2:1. The nurse interprets the result as which of the following? a. The baby's lung fields are mature. b. The mother is high risk for hemorrhage. c. The baby's kidneys are functioning poorly. d. The mother is high risk for eclampsia.

ANS: a Feedback a. An L/S ratio of 2:1 usually indicates that the fetal lungs are mature. b. L/S ratios are unrelated to maternal blood loss. c. L/S ratios are unrelated to fetal renal function. d. L/S ratios are unrelated to maternal risk for becoming eclamptic.

The laboratory reported that the L/S ratio (lecithin/sphingomyelin) results from an amniocentesis of a gravid client with preeclampsia are 2:1. The nurse interprets the result as which of the following? a. The baby's lung fields are mature. b. The mother is high risk for hemorrhage. c. The baby's kidneys are functioning poorly. d. The mother is high risk for eclampsia.

ANS: a Feedback a. An L/S ratio of 2:1 usually indicates that the fetal lungs are mature. b. L/S ratios are unrelated to maternal blood loss. c. L/S ratios are unrelated to fetal renal function. d. L/S ratios are unrelated to maternal risk for becoming eclamptic.

Which of the following is an indication for the administration of methylergonovine? a. Boggy uterus that does not respond to massage and oxytocin therapy b. Woman with a large hematoma c. Woman with a deep vein thrombosis d. Woman with severe postpartum depression

ANS: a Feedback a. Methylergonovine (methergine) is ordered for PPH due to uterine atony or subinvolution. It is used when massage and oxytocin therapy have failed to contract the uterus. b. Hematoma occurs when blood collects within the connective tissues of the vagina or perineal areas related to a vessel that ruptured and continues to bleed. Methylergonovine stimulates contraction of the smooth muscle of the uterus and would not have an effect on the vaginal or perineal areas. c. Heparin is usually prescribed for treatment of thrombosis. d. Methylergonovine is prescribed for treatment of uterine atony.

32. The perinatal nurse is providing care to Marilyn, a 25-year-old G1 TPAL 0000 woman hospitalized with severe hypertension at 33 weeks' gestation. The nurse is preparing to administer the second dose of beta-methasone prescribed by the physician. Marilyn asks: "What is this injection for again?" The nurse's best response is: a. "This is to help your baby's lungs to mature." b. "This is to prepare your body to begin the labor process." c. "This is to help stabilize your blood pressure." d. "This is to help your baby grow and develop in preparation for birth."

ANS: a Feedback a. Antenatal glucocorticoids such as beta-methasone may be given (12 mg IM 24 hours apart) to promote fetal lung maturity if the gestational age is less than 34 weeks and childbirth can be delayed for 48 hours. b. Antenatal glucocorticoids such as beta-methasone may be given (12 mg IM 24 hours apart) to promote fetal lung maturity if the gestational age is less than 34 weeks and childbirth can be delayed for 48 hours. c. Antenatal glucocorticoids such as beta-methasone may be given (12 mg IM 24 hours apart) to promote fetal lung maturity if the gestational age is less than 34 weeks and childbirth can be delayed for 48 hours. d. Antenatal glucocorticoids such as beta-methasone may be given (12 mg IM 24 hours apart) to promote fetal lung maturity if the gestational age is less than 34 weeks and childbirth can be delayed for 48 hours.

Augmentation of labor: a. Is part of the active management of labor instituted when the labor process is unsatisfactory and uterine contractions are inadequate b. Relies on more invasive methods when oxytocin and amniotomy have failed c. Is elective induction of labor d. Is an operative vaginal delivery that uses vacuum cups

ANS: a Feedback a. Augmentation stimulates uterine contractions after labor has started but not progressed appropriately. b. Augmentation uses amniotomy and oxytocin. c. Augmentation stimulates labor. d. Vacuum delivery is not part of augmentation of labor.

17. For the patient with which of the following medical problems should the nurse question a physician's order for beta agonist tocolytics? a. Type 1 diabetes mellitus b. Cerebral palsy c. Myelomeningocele d. Positive group B streptococci culture

ANS: a Feedback a. Beta agonists often elevate serum glucose levels. The nurse should question the order. b. Beta agonists are not contraindicated for patients with cerebral palsy. c. Beta agonists are not contraindicated for patients with myelomeningocele. d. Beta agonists are not contraindicated for patients with group B streptococci.

A 37-year-old gravid 8 para 7 woman was admitted to the postpartum unit at 2 hours postbirth. On admission to the unit, her fundus was U/U, midline, and firm, and her lochia was moderate rubra. An hour later, her fundus is midline and boggy, and the lochia is heavy with small clots. Based on this assessment data, the first nursing action is: a. Massage the fundus of the uterus. b. Assist the woman to the bathroom and reassess the fundus. c. Notify the physician or midwife. d. Start IV oxytocin therapy as per standing orders.

ANS: a Feedback a. Correct. Based on the assessment data that the uterus is midline and boggy, the woman is experiencing uterine atony. b. Assisting the woman to the bathroom would be a nursing action if the uterus was not midline. c. Oxytocin would be given and the primary health provider would be notified if the uterus did not respond to uterine massage. d. Oxytocin would be given and the primary health provider would be notified if the uterus did not respond to uterine massage.

6. Which of the following nursing diagnoses is of highest priority for a client with an ectopic pregnancy who has developed disseminated intravascular coagulation (DIC)? a. Risk for deficient fluid volume b. Risk for family process interrupted c. Risk for disturbed identity d. High risk for injury

ANS: a Feedback a. Correct. The client is at high risk for hypovolemia which is life threatening and takes precedence over any psychosocial or less pressing diagnoses. b. This is a psychosocial diagnosis and is not life threatening. c. This is a psychosocial diagnosis and is not life threatening. d. The client is at risk for injury; however, the diagnosis of deficient fluid volume is more descriptive and has clearly defined goals and interventions.

3. Identify the hallmark of placenta previa that differentiates it from abruptio placenta. a. Sudden onset of painless vaginal bleeding b. Board-like abdomen with severe pain c. Sudden onset of bright red vaginal bleeding d. Severe vaginal pain with bright red bleeding

ANS: a Feedback a. Correct. When the placenta attaches to the lower uterine segment near or over the cervical os, bleeding may occur without the onset of contractions or pain. b. The hallmark for abruptio placenta is pain and a board-like abdomen. c. Bright red bleeding could be related to abruptio placenta, placenta previa, or other complications of pregnancy. d. Pain is not a hallmark of placenta previa.

During the postpartum assessment, the perinatal nurse notes that a patient who has just experienced a forceps-assisted birth now has a large quantity of bright red bleeding. Her uterine fundus is firm. The nurse's most appropriate action is to notify the physician/certified nurse midwife and describe a: a. Need for vaginal assessment and repair b. Requirement for an oxytocin infusion c. Need for further information for the woman/family about forceps d. Requirement for bladder assessment and catheterization

ANS: a Feedback a. In the presence of a firm fundus and bright red bleeding, after a forceps-assisted birth there is a need for vaginal assessment and there may be a need for repair. b. The fundus is firm, and oxytocin is not indicated. c. There is no indication in this scenario that the family needs more information. d. There is no indication in this scenario that the bladder is contributing to the bleeding.

A postpartum woman has been diagnosed with postpartum psychosis. Which of the following actions should the nurse perform? a. Supervise all infant care. b. Maintain client on strict bed rest. c. Restrict visitation to her partner. d. Carefully monitor toileting.

ANS: a Feedback a. It is essential that a client diagnosed with postpartum (PP) psychosis not be left alone with her infant. b. There is no need for a client with PP psychosis to be on strict bed rest. c. Visitation is not usually restricted to the woman's partner. d. There is no need to monitor the client's toileting.

The nurse is developing a plan of care for a client who is in the "taking-in" phase after delivering a healthy baby boy. Which of the following should the nurse include in the plan? a. Provide the client with a nutritious meal. b. Teach baby care skills like diapering. c. Discuss the pros and cons of circumcision. d. Counsel her regarding future sexual encounters.

ANS: a Feedback a. Mothers are very hungry immediately after delivery. The nurse should provide the client with food. b. Baby care skills should be taught during the "taking-hold" phase. c. Baby care needs should be discussed during the "taking-hold" phase. d. A discussion of sexual issues should be deferred until the "taking-hold" phase or the "letting go" phase.

A neonate is born at 33 weeks' gestation with a birth weight of 2400 grams. This neonate would be classified as: a. Low birth weightb. Very low birth weightc. Extremely low birth weightd. Very premature

ANS: a Feedback a. Neonates with a birth weight of less than 2500 grams but greater than 1500 grams are classified as low birth weight. b. Neonates with birth weight less than 1500 grams but greater than 1000 grams are classified as very low birth weight. c. Neonates with birth weight less than 1000 grams are classified as extremely low birth weight. d. Neonates born less than 32 weeks' gestation are classified as very premature.

The perinatal nurse notes a rapid decrease in the fetal heart rate that does not recover immediately following an amniotomy. The most likely cause of this obstetrical emergency is: a. Prolapsed umbilical cord b. Vasa previa c. Oligohydramnios d. Placental abruption

ANS: a Feedback a. The nurse needs to assess the fetal heart rate immediately before and after the artificial rupture of membranes. Changes such as transient fetal tachycardia may occur and are common. However, other FHR patterns such as bradycardia and variable decelerations may be indicative of cord compression or prolapse. b. Vasa previa is abnormal insertion of the cord into the placenta c. Oligohydramnios is a decreased amount of amniotic fluid. d. Placenta abruption is separation of the placenta from the uterine wall. In this scenario, prolapsed cord is the most likely cause of the abrupt deceleration in the FHR.

The physician has ordered intravenous oxytocin for induction for four gravidas. In which of the following situations should the nurse refuse to comply with the order? a. Primigravida with complete placenta previa b. Multigravida with extrinsic asthma c. Primigravida who is 38 years old d. Multigravida who is colonized with group B streptococci

ANS: a Feedback a. The nurse should refuse to comply with this order because labor is contraindicated for a patient with complete placenta previa. This patient will have to be delivered via cesarean section. b. Induction is not contraindicated for patients with asthma. c. Induction is not contraindicated for patients who are 38 years old. d. Induction is not contraindicated for patients with group B streptococci.

A post-cesarean birth woman has been diagnosed with paralytic ileus. Which of the following symptoms would the nurse expect to see? a. Abdominal distension b. Polyuria c. Diastasis recti d. Dependent edema

ANS: a Feedback a. The nurse would expect to see a distended abdomen in a client with a paralytic ileus. b. Polyuria is unrelated to a paralytic ileus. c. Diastasis recti is unrelated to a paralytic ileus. d. Dependent edema is unrelated to a paralytic ileus.

28. A primiparous woman has been admitted at 35 weeks' gestation and diagnosed with HELLP syndrome. Which of the following laboratory changes is consistent with this diagnosis? a. Hematocrit dropped to 28%. b. Platelets increased to 300,000 cells/mm3. c. Red blood cells increased to 5.1 million cells/mm3. d. Sodium dropped to 132 mEq/dL.

ANS: a Feedback a. The nurse would expect to see a drop in the hematocrit: The H in HELLP stands for hemolysis. b. The nurse would expect to see low platelets. c. The nurse would expect to see hemolysis. d. The sodium is usually unaffected in HELLP syndrome.

15. Which of the following signs or symptoms would the nurse expect to see in a woman with concealed abruptio placentae? a. Increasing abdominal girth measurements b. Profuse vaginal bleeding c. Bradycardia with an aortic thrill d. Hypothermia with chills

ANS: a Feedback a. The nurse would expect to see increasing abdominal girth measurements. b. Profuse vaginal bleeding is rarely seen in placental abruption and is never seen when the abruption is concealed. c. With excessive blood loss, the nurse would expect to see tachycardia. d. The nurse would expect to see a stable temperature.

A 16-year-old woman delivers a healthy, full-term male infant. The nurse notes the following behaviors 2 hours after the birth: Woman holds baby away from her body; woman refers to baby as "he"; woman verbalizes she wanted a baby girl; woman requests that baby be placed in the bassinet so she can eat her lunch. The most appropriate nursing diagnosis for this woman is: a. At risk for impaired parenting related to disappointment with baby as evidenced by verbalizing she wanted a girl b. At risk for impaired parenting related to nonnurturing behaviors as evidenced by holding baby away from body c. At risk for impaired mother-infant attachment as evidenced by woman requesting baby being placed in bassinet d. At risk for impaired mother-infant attachment related to disappointment as evidenced by calling baby "he"

ANS: a Feedback a. The potential is for impaired parenting related to disappointment in the gender of the baby. b. Holding baby away from her body during the first few hours is part of the maternal touch process. c. Focusing on eating during the first few hours is a behavior of taking-in and is anticipated during this phase. d. Some parents have not selected a name for their baby and will refer to their baby as "he" or "she." There is concern if the woman calls her baby "it."

The nurse is assisting a physician in the delivery of a baby via vacuum extraction. Which of the following nursing diagnoses for the gravida is appropriate at this time? a. Risk for injury b. Colonic constipation c. Risk for impaired parenting d. Ineffective individual coping

ANS: a Feedback a. There is a risk for injury. For example, the patient could suffer a cervical, vaginal, or perineal laceration. b. A diagnosis of colonic constipation is unrelated to the fact that the baby was delivered by forceps. c. There is nothing in the scenario that implies that the patient is at risk for impaired parenting. d. There is nothing in the scenario that implies that the patient is at risk for ineffective individual coping.

A baby boy was just born to a mother who had positive vaginal cultures for group B streptococci. The mother was admitted to the labor room 30 minutes before the birth. For which of the following should the nursery nurse closely observe this baby? a. Grunting b. Acrocyanosis c. Pseudostrabismus d. Hydrocele

ANS: a Feedback a. This infant is high risk for respiratory distress. The nurse should observe this baby carefully for grunting. b. Acrocyanosis is a normal finding. c. Pseudostrabismus is a normal finding. d. Hydrocele should be reported to the neonatologist. It is not, however, an emergent problem, and it is not related to group B streptococci colonization in the mother.

The perinatal nurse teaches the postpartum woman about warning signs regarding development of postpartum infection. Signs and symptoms that merit assessment by the health-care provider include the development of a fever and: a. Breast engorgement b. Uterine tenderness c. Diarrhea d. Emotional lability

ANS: b During the immediate postpartum period, the most common site of infection is the uterine endometrium. This infection presents with a temperature elevation over 101°F, often within the first 24 to 48 hours after childbirth, followed by uterine tenderness and foul-smelling lochia.

Tanya, a 30-year-old woman, is being prepared for an elective cesarean birth. The perinatal nurse assists the anesthesiologist with the spinal block and then positions Tanya in a supine position. Tanya's blood pressure drops to 90/52, and there is a decrease in the fetal heart rate to 110 bpm. The perinatal nurse's best response is to: a. Place a wedge under Tanya's left hip. b. Discontinue Tanya's intravenous administration. c. Have naloxone (Narcan) ready for administration. d. Have epinephrine ready for administration.

ANS: a In the event of severe maternal hypotension, the nurse should place the patient in a lateral position or use a wedge under the hip to displace the uterus, elevate the legs, maintain or increase the IV infusion rate, and administer oxygen by face mask at 10 to 12 L/min, or according to institution protocol.

36. Metabolic changes during pregnancy __________ glucose tolerance. a. lower b. increase c. maintain d. alter

ANS: a Metabolic changes during pregnancy lower glucose tolerance.

The perinatal nurse recognizes that a risk factor for postpartum depression is: a. Inadequate social support b. Age >35 years c. Gestational hypertension d. Regular schedule of prenatal care

ANS: a Recognized risk factors for postpartum depression include an undesired or unplanned pregnancy, a history of depression, recent major life changes such as the death of a family member, moving to a new community, lack of family or social support, financial stress, marital discord, adolescent age, and homelessness.

35. Your antepartal patient is 38 weeks' gestation, has a history of thrombosis, and has been on strict bed rest for the last 12 hours. She is now experiencing shortness of breath. What about the patient may be a contributing factor for her shortness of breath? a. Physiologic changes in pregnancy result in vasodilation, which increases the tendency to form blood clots. b. Physiologic changes in pregnancy result in vasoconstriction, which increases the tendency to form blood clots. c. Physiologic changes in pregnancy result in anemia, which increases the tendency to form blood clots. d. Physiologic changes in pregnancy result in decreased perfusion to the lungs, which increases the tendency to form blood clots.

ANS: a The patient's shortness of breath, bed rest, and history of thrombosis indicate possible pulmonary embolism. Her pregnant state also increases the potential for thrombosis resulting from increased levels of coagulation factors and decreased fibrinolysis, venous dilation, and obstruction of the venous system by the gravid uterus. Thromboembolitic diseases occurring most frequently in pregnancy include deep vein thrombosis and pulmonary embolism.

The postpartum nurse caring for a 20-year-old G1 P0 woman who 3 hours ago delivered a healthy full-term infant, observes the woman who is lightly touching her baby girl with her fingertips but who seems to be uncomfortable holding her baby close to her body. Which of the following is an accurate interpretation of these observed behaviors? a. The woman is in the initial stage of maternal touch. b. The woman is in the taking-in phase. c. The woman is having difficulty in bonding with her baby. d. The woman needs to be medicated for pain.

ANS: a These are classical signs of the initial stage of Rubin's maternal touch.

The perinatal nurse teaches the student nurse that deep breathing exercises following a cesarean birth are critical to the prevention of (select all that apply): a. Pneumonia b. Atelectasis c. Abdominal distension d. Increased tidal volume

ANS: a, b Incisional pain and abdominal distension often cause patients to adopt shallow breathing patterns that can lead to decreased gas exchange and a reduced tidal volume. To facilitate adequate lung functions, patients should be taught how to perform pulmonary exercises. Expectoration of secretions and deep breathing help prevent common complications including atelectasis and pneumonia. Abdominal distension and gas pains are common after abdominal surgery and result from delayed peristalsis

20. Nursing actions that decrease the risk of skin breakdown include which of the following? (Select all that apply.) a. Using gelled mattresses b. Using emollients in groin and thigh areas c. Using transparent dressings d. Drying thoroughly

ANS: a, b, c a. Use of gelled mattresses decreases the risk of pressure sores. b. Use of emollients reduces the risk of irritation from urine. c. Use of transparent dressings reduces the risk of friction injuries. d. Drying thoroughly is important in maintaining body heat.

Documentation related to vacuum delivery includes which of the following: a. Fetal heart rate b. Timing and number of applications c. Position and station of fetal head d. Maternal position

ANS: a, b, c Assessment of fetal heart rate is part of second-stage management, timing and number of applications are part of standard of care related to safe vacuum deliveries, and position and station of fetal head are noted for safe vacuum extraction. Maternal position is not critical to the documentation related to vacuum deliveries.

Hyperstimulation is defined as: a. Contractions lasting more than 2 minutes b. Five or more contractions in 10 minutes c. Contractions occurring within 1 minute of each other d. Uterine resting tone below 20 mm/Hg

ANS: a, b, c Contractions lasting more than 2 minutes, five or more contractions in 10 minutes, and contractions occurring within 1 minute of each other describe the criteria for hyperstimulation. Uterine resting tone below 20 mm/Hg reflects normal uterine resting tone.

A G2 P1 woman who experienced a prolonged labor and prolonged rupture of membranes is at risk for metritis. Which of the following nursing actions are directed at decreasing this risk? (Select all that apply.) a. Instruct woman to increase her fluid intake b. Instruct woman to change her peri-pads after each voiding c. Instruct woman to ambulate in the halls four times a day d. Instruct woman to apply ice packs to the perineum

ANS: a, b, c Feedback a. Maintaining adequate hydration can decrease a person's risk for infection. b. Lochia is a media for bacterial growth, so it is important to frequently change the peri-pads. c. Ambulation can decrease the risk of infection by promoting uterine drainage. d. Ice pack therapy is directed at decreasing edema of the perineum and promoting comfort. It has no effect on metriosis.

Which of the following nursing actions can assist a man in his transition to fatherhood? (Select all that apply.) a. Ask the man to share his ideas of what it means to be a father. b. Demonstrate infant care such as diapering and feeding. c. Engage couple in a discussion regarding each other's expectations of the fathering role. d. Provide the man with information on infant care.

ANS: a, b, c, d Each of these actions can assist the father in his transition. It is important for the man to be able to learn and practice infant care skills in a nonthreatening environment. It is also important for the man to be able to openly talk about his feelings regarding fatherhood and for the couple to identify mutual expectations of the fathering role.

43. The perinatal nurse describes risk factors for placenta previa to the student nurse. Placenta previa risk factors include (select all that apply): a. Cocaine use b. Tobacco use c. Previous caesarean birth d. Previous use of medroxyprogesterone (Depo-Provera)

ANS: a, b, c Feedback a. Placenta previa may be associated with risk factors including smoking, cocaine use, a prior history of placenta previa, closely spaced pregnancies, African or Asian ethnicity, and maternal age greater than 35 years. b. Placenta previa may be associated with risk factors including smoking, cocaine use, a prior history of placenta previa, closely spaced pregnancies, African or Asian ethnicity, and maternal age greater than 35 years. c. Placenta previa may be associated with conditions that cause scarring of the uterus such as a prior cesarean section, multiparity, or increased maternal age. d. Previous use of medroxyprogesterone (Depo-Provera) is not a risk factor for placenta previa.

46. The perinatal nurse provides a hospital tour for couples and families preparing for labor and birth in the future. Teaching is an important component of the tour. Information provided about preterm labor and birth prevention includes (select all that apply): a. Encouraging regular, ongoing prenatal care b. Reporting symptoms of urinary frequency and burning to the health-care provider c. Coming to the labor triage unit if back pain or cramping persist or become regular d. Lying on the right side, withholding fluids, and counting fetal movements if contractions occur every 5 minutes

ANS: a, b, c Feedback a. The nurse should encourage all pregnant women to obtain prenatal care and screen for vaginal and urogenital infections and treat appropriately, and remind pregnant women to call their provider repeatedly if symptoms of preterm labor occur. b. Educating all women of childbearing age about preterm labor is a crucial component of prevention. The nurse should encourage all pregnant women to obtain prenatal care and screen for vaginal and urogenital infections and treat appropriately, and remind pregnant women to call their provider repeatedly if symptoms of preterm labor occur. c. Educating all women of childbearing age about preterm labor is a crucial component of prevention. The nurse should encourage all pregnant women to obtain prenatal care and screen for vaginal and urogenital infections and treat appropriately, and remind pregnant women to call their provider if symptoms of preterm labor occur. d. Lying on the right side; drinking fluids, not withholding fluids; and counting fetal movements if contractions occur every 5 minutes are recommended if a woman thinks she is contracting.

Nursing actions that decrease the risk of skin breakdown include which of the following? (Select all that apply.) a. Using gelled mattresses b. Using emollients in groin and thigh areas c. Using transparent dressings d. Drying thoroughly

ANS: a, b, c Feedback a. Use of gelled mattresses decreases the risk of pressure sores. b. Use of emollients reduces the risk of irritation from urine. c. Use of transparent dressings reduces the risk of friction injuries. d. Drying thoroughly is important in maintaining body heat.

Which of the following actions can decrease the risk for a postpartum infection? (Select all that apply.) a. Diet high in protein and vitamin C b. Increased fluid intake c. Ambulating within a few hours after delivery d. Washing nipples with soap prior to each breastfeeding session

ANS: a, b, c Protein and vitamin C assist with tissue healing. Rehydrating a woman after delivery can assist with decreasing risk for infections. Early ambulation decreases risk for infection by promoting uterine drainage. The woman should not wash her breasts with soap because soap can dry the tissue and increase the woman's risk for tissue breakdown.

49. Marked hemodynamic changes in pregnancy can impact the pregnant woman with cardiac disease. Signs and symptoms of deteriorating cardiac status include (select all that apply): a. Orthopnea b. Nocturnal dyspnea c. Palpitations d. Irritation

ANS: a, b, c Signs and symptoms of deteriorating cardiac status with cardiac disease include orthopnea, nocturnal dyspnea, and palpitations, but do not include irritation.

Which of the following are primary risk factors for subinvolution of the uterus? (Select all that apply.) a. Fibroids b. Retained placental tissue c. Metritis d. Urinary tract infection

ANS: a, b, c Uterine fibroids can interfere with involution. Retained placental tissue does not allow the uterus to remain contracted. Infection in the uterus is a risk factor for subinvolution. UTI does not interfere with involution of the uterus.

25. A nurse is completing the initial assessment on a neonate of a mother with type I diabetes. Important assessment areas for this neonate include which of the following? (Select all that apply.) a. Assessment of cardiovascular system b. Assessment of respiratory system c. Assessment of musculoskeletal system d. Assessment of neurological system

ANS: a, b, c, d a. Neonates of mothers with type I diabetes are at higher risk for cardiac anomalies. b. Neonates of mothers with type I diabetes are at higher risk for RDS due to a delay in surfactant production related to high maternal glucose levels. c. Neonates of mothers with type I diabetes are usually large and are at risk for a fractured clavicle. d. Neonates of mothers with type I diabetes are at higher risk for neurological damage and seizures due to neonatal hyperinsulinism.

23. Which of the following are common assessment findings of postmature neonates? (Select all that apply.) a. Dry and peeling skin b. Abundant vernix caseosa c. Hypoglycemia d. Thin, wasted appearance

ANS: a, b, c, d a. Vernix caseosa covers the fetus's body around 17 to 20 weeks' gestation; as pregnancy advances, the amount of vernix decreases. Vernix prevents water loss from the skin to the amniotic fluid; as the amount of vernix decreases, an increasing amount of water is lost from the skin. This contributes to the dry and peeling skin seen in postmature neonates. b. Vernix caseosa covers the fetus's body around 17 to 20 weeks' gestation; as pregnancy advances, the amount of vernix decreases. c. Placental insufficiency related to the aging of the placenta may result in postmaturity syndrome, in which the fetus begins to use its subcutaneous fat stores and glycemic stores. This results in the thin and wasted appearance of the neonate and risk for hypoglycemia during the first few hours post-birth. d. Placental insufficiency related to the aging of the placenta may result in postmaturity syndrome, in which the fetus begins to use its subcutaneous fat stores and glycemic stores. This results in the thin and wasted appearance of the neonate and risk for hypoglycemia during the first few hours post-birth.

The NICU nurse's patient assignment includes an infant who is 25 weeks' gestation. The nurse knows that according to the gestational age, this infant would be described as __________.

ANS: very premature The definition of very premature is a neonate born at less than 32 weeks' gestation. The definition of premature is a neonate born between 32 and 34 weeks' gestation. The definition of late premature is a neonate born between 34 and 37 weeks' gestation.

A nurse is completing the initial assessment on a neonate of a mother with type I diabetes. Important assessment areas for this neonate include which of the following? (Select all that apply.) a. Assessment of cardiovascular system b. Assessment of respiratory system c. Assessment of musculoskeletal system d. Assessment of neurological system

ANS: a, b, c, d Feedback a. Neonates of mothers with type I diabetes are at higher risk for cardiac anomalies. b. Neonates of mothers with type I diabetes are at higher risk for RDS due to a delay in surfactant production related to high maternal glucose levels. c. Neonates of mothers with type I diabetes are usually large and are at risk for a fractured clavicle. d. Neonates of mothers with type I diabetes are at higher risk for neurological damage and seizures due to neonatal hyperinsulinism.

Which of the following are common assessment findings of postmature neonates? (Select all that apply.) a. Dry and peeling skin b. Abundant vernix caseosa c. Hypoglycemia d. Thin, wasted appearance

ANS: a, b, c, d Feedback a. Vernix caseosa covers the fetus's body around 17 to 20 weeks' gestation; as pregnancy advances, the amount of vernix decreases. Vernix prevents water loss from the skin to the amniotic fluid; as the amount of vernix decreases, an increasing amount of water is lost from the skin. This contributes to the dry and peeling skin seen in postmature neonates. b. Vernix caseosa covers the fetus's body around 17 to 20 weeks' gestation; as pregnancy advances, the amount of vernix decreases. c. Placental insufficiency related to the aging of the placenta may result in postmaturity syndrome, in which the fetus begins to use its subcutaneous fat stores and glycemic stores. This results in the thin and wasted appearance of the neonate and risk for hypoglycemia during the first few hours post-birth. d. Placental insufficiency related to the aging of the placenta may result in postmaturity syndrome, in which the fetus begins to use its subcutaneous fat stores and glycemic stores. This results in the thin and wasted appearance of the neonate and risk for hypoglycemia during the first few hours post-birth.

A nurse is caring for a woman who is 4 hours post-cesarean birth for arrest of labor. The labor and operative records indicate that she had premature rupture of membranes followed by 36 hours of labor. Her IV fluid intake for the past 24 hours is 2500 mL. The estimated blood loss is 1500 mL. Based on this data, the woman is at risk for which of the following? (Select all that apply.) a. Fluid volume deficit b. Infection c. Impaired mother-infant attachment d. Falls

ANS: a, b, c, dThe woman is at risk for fluid volume deficit related to blood loss and risk for postpartum hemorrhage due to risk of uterine atony. She is at risk for infection related to premature and prolonged rupture of membranes. The woman is at risk for impaired mother-infant attachment related to maternal pain and exhaustion. She is at risk for falls related to anesthesia and orthostatic hypotension.

24. A nurse is caring for a 40 weeks' gestation neonate. The neonate is 12 hours post-birth and has been admitted to the NICU for meconium aspiration. The nurse recalls that the following are potential complications related to meconium aspiration (select all that apply): a. Obstructed airway b. Hyperinflation of the alveoli c. Hypoinflation of the alveoli d. Decreased surfactant proteins

ANS: a, b, d a. The presence of meconium in the neonate's lungs can cause a partial obstruction of the lower airway that leads to a trapping of air and a hyperinflation of the alveoli. b. The presence of meconium in the neonate's lungs can cause a partial obstruction of the lower airway that leads to a trapping of air and a hyperinflation of the alveoli. c. The presence of meconium in the neonate's lungs can cause a partial obstruction of the lower airway that leads to a trapping of air and a hyperinflation of the alveoli. d. The presence of meconium in the lungs can also cause a chemical pneumonitis and inhibit surfactant production.

Which of the following factors place a new mother at risk for parenting? (Select all that apply.) a. She is 17 years old. b. Family income is below the average income. c. Her parents live in the same city and are perceived as helpful. d. She dropped out of school at age 13.

ANS: a, b, d Adolescent parents may have a more difficult transition to parenthood because they have not made the transition to adulthood. Financial concerns can hamper the transition to parenthood because the focus of attention may be on where to get money to pay for daily living expenses versus on the care of their newborn. Decreased ability to read and comprehend information regarding child care may hamper the ability to gain knowledge about the care of their child.

48. Betamethasone is a steroid that is given to a pregnant woman with signs of preterm labor. The purpose of giving steroids is to (select all that apply): a. Stimulate the production of surfactant in the preterm infant b. Be given between 24 and 34 weeks' gestation c. Increase the severity of respiratory distress d. Accelerate fetal lung maturity

ANS: a, b, d Betamethasone is a steroid that is given to pregnant women with signs of preterm labor between 24 and 34 weeks' gestation. It stimulates the production of surfactant in the preterm infant and accelerates fetal lung maturity.

47. The perinatal nurse describes for the new nurse the various risks associated with prolonged premature preterm rupture of membranes. These risks include (select all that apply): a. Chorioamnionitis b. Abruptio placentae c. Operative birth d. Cord prolapse

ANS: a, b, d Even though maintaining the pregnancy to gain further fetal maturity can be beneficial, prolonged PPROM has been correlated with an increased risk of chorioamnionitis, placental abruption, and cord prolapse.

A nurse is caring for a 40 weeks' gestation neonate. The neonate is 12 hours post-birth and has been admitted to the NICU for meconium aspiration. The nurse recalls that the following are potential complications related to meconium aspiration (select all that apply): a. Obstructed airway b. Hyperinflation of the alveoli c. Hypoinflation of the alveoli d. Decreased surfactant proteins

ANS: a, b, d Feedback a. The presence of meconium in the neonate's lungs can cause a partial obstruction of the lower airway that leads to a trapping of air and a hyperinflation of the alveoli. b. The presence of meconium in the neonate's lungs can cause a partial obstruction of the lower airway that leads to a trapping of air and a hyperinflation of the alveoli. c. The presence of meconium in the neonate's lungs can cause a partial obstruction of the lower airway that leads to a trapping of air and a hyperinflation of the alveoli. d. The presence of meconium in the lungs can also cause a chemical pneumonitis and inhibit surfactant production.

44. Kerry, a 30-year-old G3 TPAL 0110 woman presents to the labor unit triage with complaints of lower abdominal cramping and urinary frequency at 30 weeks' gestation. An appropriate nursing action would be to (select all that apply): a. Assess the fetal heart rate b. Obtain urine for culture and sensitivity c. Assess Kerry's blood pressure and pulse d. Palpate Kerry's abdomen for contractions

ANS: a, b, d Feedback a. Women experiencing preterm labor may complain of backache, pelvic aching, menstrual-like cramps, increased vaginal discharge, pelvic pressure, urinary frequency, and intestinal cramping with or without diarrhea. The patient's abdomen should be palpated to assess for contractions, and the fetus's heart rate should be monitored. b. Women experiencing preterm labor may complain of backache, pelvic aching, menstrual-like cramps, increased vaginal discharge, pelvic pressure, urinary frequency, and intestinal cramping with or without diarrhea. A urinalysis and urine culture and sensitivity (C & S) should be obtained on all patients who present with signs of preterm labor, and the nurse must remember that signs of UTI often mimic normal pregnancy complaints (i.e., urgency, frequency). The patient's abdomen should be palpated to assess for contractions, and the fetus's heart rate should be monitored. c. Assessment of blood pressure and pulse is not an important nursing action in this scenario. d. Women experiencing preterm labor may complain of backache, pelvic aching, menstrual-like cramps, increased vaginal discharge, pelvic pressure, urinary frequency, and intestinal cramping with or without diarrhea. The patient's abdomen should be palpated to assess for contractions and the fetus's heart rate should be monitored.

Which of the following nursing actions are directed at promoting bonding? (Select all that apply.) a. Providing opportunity for parents to hold their newborn as soon as possible following the birth. b. Providing opportunities for the couple to talk about their birth experience and about becoming parents. c. Promoting rest and comfort by keeping the newborn in the nursery at night. d. Providing positive comments to parents regarding their interactions with their newborn.

ANS: a, b, d Parent bonding can be delayed by prolonged periods of separation from their child. The other three actions support parent bonding with their newborn.

18. A nurse is caring for a 2-day-old neonate who was born at 31 weeks' gestation. The neonate has a diagnosis of respiratory distress syndrome (RDS). Which of the following medical treatments would the nurse anticipate for this neonate? (Select all that apply.) a. Exogenous surfactant b. Corticosteroids c. Continuous positive airway pressure (CPAP) d. Bronchodilators

ANS: a, c a. This is a common medical treatment for RDS. b. Corticosteroids are given to women in preterm labor to decrease the risk of RDS. c. CPAP is used to assist neonates with RDS. d. Bronchodilators are given to neonates with bronchopulmonary dysplasia (BPD).

A nurse is caring for a 2-day-old neonate who was born at 31 weeks' gestation. The neonate has a diagnosis of respiratory distress syndrome (RDS). Which of the following medical treatments would the nurse anticipate for this neonate? (Select all that apply.) a. Exogenous surfactant b. Corticosteroids c. Continuous positive airway pressure (CPAP) d. Bronchodilators

ANS: a, c Feedback a. This is a common medical treatment for RDS. b. Corticosteroids are given to women in preterm labor to decrease the risk of RDS. c. CPAP is used to assist neonates with RDS. d. Bronchodilators are given to neonates with bronchopulmonary dysplasia (BPD).

22. A nurse is caring for a 10-day-old neonate who was born at 33 weeks' gestation. Which of the following actions assist the nurse in assessing for signs of feeding tolerance? (Select all that apply.) a. Check for presence of bowel sounds b. Assess temperature c. Check gastric residual by aspirating stomach contents d. Assess stools

ANS: a, c, d a. Feedings should be held and physician notified if bowel sounds are absent. b. The neonate's temperature has no direct effect on feeding tolerance. c. Aspirated stomach contents are assessed for amount, color, and consistency. This assists in the evaluation of the degree of digestion and absorption. d. Stools are assessed for consistency, amount, and frequency. This assists in the evaluation of the degree of digestion and absorption.

Nursing actions focused at reducing a postpartum woman's risk for cystitis include which of the following? (Select all that apply.) a. Voiding within a few hours post-birth b. Oral intake of a minimum of 1000 mL per day c. Changing peri-pads every 3 to 4 hours or more frequently as indicated d. Reminding the woman to void every 3 to 4 hours while awake

ANS: a, c, d Early voiding helps flush bacteria from the urethra. Voiding every 3 to 4 hours will decrease the risk of bacterial growth in the bladder. Soiled peri-pads are a media for bacterial growth. It is recommend that a postpartum woman drink a minimum of 3000 mL/day to help dilute urine and promote frequent voiding.

A nurse is caring for a 10-day-old neonate who was born at 33 weeks' gestation. Which of the following actions assist the nurse in assessing for signs of feeding tolerance? (Select all that apply.) a. Check for presence of bowel sounds b. Assess temperature c. Check gastric residual by aspirating stomach contents d. Assess stools

ANS: a, c, d Feedback a. Feedings should be held and physician notified if bowel sounds are absent. b. The neonate's temperature has no direct effect on feeding tolerance. c. Aspirated stomach contents are assessed for amount, color, and consistency. This assists in the evaluation of the degree of digestion and absorption. d. Stools are assessed for consistency, amount, and frequency. This assists in the evaluation of the degree of digestion and absorption.

The nurse is caring for a postpartum woman who gave birth to a healthy, full-term baby girl. She has a 2-year-old son. She voices concern about her older child's adjustment to the new baby. Nursing actions that will facilitate the older son's adjustment to having a new baby in the house would include which of the following? (Select all that apply.) a. Explain to the mother that she can have her son lie in bed with her when he is visiting her in the hospital. b. Teach her son how to change the baby's diapers. c. Assist her son in holding his new baby sister. d. Recommend that she spend time reading to her older son while he sits in her lap.

ANS: a, c, d Two-year-olds enjoy being close to their mothers, including lying next to their mothers or being held. Changing diapers is not viewed as a pleasurable experience and is not developmentally appropriate for a 2-year-old. Children enjoy being able to hold their sibling and feeling "grown up."

19. Which of the following factors increases the risk of necrotizing enterocolitis (NEC) in very premature neonates? (Select all that apply.) a. Early oral feedings with formula b. Prolonged use of mechanical ventilation c. Hyperbilirubinemia d. Nasogastic feedings

ANS: a, d a. Preterm neonates have a decreased ability to digest and absorb formula. Undigested formula can cause a blockage in the intestines leading to necrosis of the bowel. b. Preterm neonates are predisposed to NEC due to alteration in blood flow to the intestines, impaired gastrointestinal host defense, and alteration in inflammatory response. c. Preterm neonates are predisposed to NEC due to alteration in blood flow to the intestines, impaired gastrointestinal host defense, and alteration in inflammatory response. d. Bacterial colonization in the intestines can occur from contaminated feeding tubes causing an inflammatory response in the bowel.

Contraindications for induction of labor include: a. Abnormal fetal position b. Postdated pregnancy c. Pregnancy-induced hypertension d. Placental abnormalities

ANS: a, d Contraindications for induction of labor include abnormal fetal position because of the risk of fetal injury and placental abnormalities because of the risk of hemorrhage. Pregnancy-induced hypertension and placental abnormalities are two of the common indications for induction of labor.

Which of the following factors increases the risk of necrotizing enterocolitis (NEC) in very premature neonates? (Select all that apply.) a. Early oral feedings with formula b. Prolonged use of mechanical ventilation c. Hyperbilirubinemia d. Nasogastic feedings

ANS: a, d Feedback a. Preterm neonates have a decreased ability to digest and absorb formula. Undigested formula can cause a blockage in the intestines leading to necrosis of the bowel. b. Preterm neonates are predisposed to NEC due to alteration in blood flow to the intestines, impaired gastrointestinal host defense, and alteration in inflammatory response. c. Preterm neonates are predisposed to NEC due to alteration in blood flow to the intestines, impaired gastrointestinal host defense, and alteration in inflammatory response. d. Bacterial colonization in the intestines can occur from contaminated feeding tubes causing an inflammatory response in the bowel.

14. Which of the following neonatal signs or symptoms would the nurse expect to see in a neonate with an elevated bilirubin level? a. Low glucose b. Poor feeding c. Hyperactivity d. Hyperthermia

ANS: b a. Hypoglycemia is not a sign that is related to an elevated bilirubin level. b. The baby is likely to feed poorly. An elevated bilirubin level adversely affects the central nervous system. Babies are often sleepy and feed poorly when the bilirubin level is elevated. c. Hyperactivity is the opposite of the behavior one would expect the baby to exhibit. d. Hyperthermia is not directly related to an elevated bilirubin level.

3. A full-term neonate who is 30 hours old has a bilirubin level of 10 mg/dL. The neonate has a yellowish tint to the skin of the face. The mother is breastfeeding her newborn. The nurse caring for this neonate would anticipate which of the following interventions? a. Phototherapy b. Feeding neonate every 2 to 3 hours c. Switch from breastfeeding to bottle feeding d. Assess red blood cell count

ANS: b a. Phototherapy is considered when the levels are 12 mg/dL or higher when the neonate is 25 to 48 hours old. Neonates re-absorb increased amounts of unconjugated bilirubin in the intestines due to lack of intestinal bacteria and decreased gastrointestinal motility. b. Adequate hydration promotes excretion of bilirubin in the urine. c. Colostrum acts as a laxative and assists in the passage of meconium. d. Assessing RBC is not a treatment for hyperbilirubinemia.

The perinatal nurse observes the new mother watching her baby daughter closely, touching her face, and asking many questions about infant feeding. This stage of mothering is best described as: a. Taking in b. Taking hold c. Taking charge d. Taking time

ANS: b As the mother's physical condition improves, she begins to take charge and enters the taking-hold phase where she assumes care for herself and her infant. At this time, the mother eagerly wants information about infant care and shows signs of bonding with her infant. During this phase, the nurse should closely observe mother-infant interactions for signs of poor bonding, and if present, implement actions to facilitate attachment.

Karen, a G2, P1, experienced a precipitous birth 90 minutes ago. Her infant is 4200 grams and a repair of a second-degree laceration was needed following the birth. As part of the nursing assessment, the nurse discovers that Karen's uterus is boggy. Furthermore, it is noted that Karen's vaginal bleeding has increased. The nurse's most appropriate first action is to: a. Assess vital signs including blood pressure and pulse. b. Massage the uterine fundus with continual lower segment support. c. Measure and document each perineal pad changed in order to assess blood loss. d. Ensure appropriate lighting for a perineal repair if it is needed.

ANS: b As the primary caregiver, the registered nurse may be the first person to identify excessive blood loss and initiate immediate actions. The nurse should first locate the uterine fundus and initiate fundal massage. Nursing actions performed after the massage are frequent vital sign measurements with an automatic device, measuring the length of time it takes for blood loss to saturate a pad, and assessing for bladder distention.

The perinatal nurse understands that the purpose of combining an opioid with a local anesthetic agent in an epidural is primarily to: a. Increase the total anesthetic volume b. Preserve a greater amount of maternal motor function c. Increase the intensity of the motor and sensory block d. Decrease the number of side effects

ANS: b Combining an opioid with a local anesthetic agent reduces the total amount of anesthetic required and helps to preserve a greater amount of maternal motor function.

The perinatal nurse is preparing a woman for a scheduled cesarean birth. The woman will be receiving spinal anesthesia for the birth. In order to prevent maternal hypotension, the nurse: a. Assists the woman to lie down in a supine position. b. Administers a rapid intravenous infusion of 500 mL of normal saline. c. Assesses blood pressure and pulse every 5 minutes, three times, before the spinal insertion. d. Encourages frequent cleansing breaths after the patient has been placed in the correct position for the anesthesia administration.

ANS: b Complications that may occur with spinal anesthesia block include maternal hypotension, decreased placental perfusion, and an ineffective breathing pattern. Prior to administration, the patient's fluid balance is assessed, and IV fluids are administered to reduce the potential for sympathetic blockade (decreased cardiac output that results from vasodilation with pooling of blood in the lower extremities). Following administration of the anesthetic, the patient's blood pressure, pulse, and respirations and fetal heart rate must be taken and documented every 5 to 10 minutes.

The nurse notes that a new father gazes at his baby for prolonged periods of time and comments that his baby is beautiful and he is very happy having a baby. These behaviors are commonly associated with: a. Bonding b. Engrossment c. Couvade syndrome d. Attachment

ANS: b Feedback a. Bonding is defined as the emotional feelings that begin during pregnancy or shortly after birth between the parent and the newborn. Bonding is unidirectional from parent to newborn. b. Correct. Characteristics of engrossment are visual awareness of baby, tactile awareness of baby, perception that baby is perfect, strong attraction to baby, feeling of strong elation, and increased self-esteem. c. Couvade syndrome relates to a set of pregnancy symptoms the father experiences during pregnancy of the woman. d. Attachment is a connection that forms from parent to infant and infant to parent. Attachment has a lifelong impact on the developing individual.

You are caring for a primiparous woman admitted to labor and delivery for induction of labor at 42 weeks' gestation. She asks you to explain the factors that contribute to prolonged labor. The best response would be to state the following: a. Primiparous women are not at risk for dystocia because they usually have small babies. b. Dystocia is related to uterine contractions, the pelvis, the fetus, the position of the mother, and psychosocial response. c. Labor is primarily associated with pelvic abnormalities. d. Dystocia is typically diagnosed prior to labor based on pelvimetry.

ANS: b Feedback a. Dystocia is not exclusively related to fetal size and being primiparous. b. This is the only correct definition of prolonged labor and dystocia. The success of any labor depends on the complex interrelationship of several factors: fetal size, presentation, position, size and shape of the pelvis, and quality of uterine contractions. c. Pelvic abnormality is the least important contributor to dystocia. d. Dystocia is diagnosed during, not prior to, labor.

9. While educating the client with class II cardiac disease, at 28 weeks' gestation, the nurse instructs the client to notify the physician if she experiences which of the following conditions? a. Emotional stress at work b. Increased dyspnea while resting c. Mild pedal and ankle edema d. Weight gain of 1 pound in 1 week

ANS: b Feedback a. Emotional stress increases cardiac workload; however, without symptoms of cardiac decompensation, this is not immediately concerning. b. Increasing dyspnea, at rest, can be a sign of cardiac decompensation leading to increased congestive heart failure. c. Mild edema during the third trimester is normal. However, increasing edema and pitting edema should be reported as they can be a sign of increasing CHF. d. A weight gain of 1 pound per week is expected during the third trimester.

Your pregnant patient is in her first trimester and is scheduled for an abdominal ultrasound. When explaining the rationale for early pregnancy ultrasound, the best response is: a. "The test will help to determine the baby's position." b. "The test will help to determine how many weeks you are pregnant." c. "The test will help to determine if your baby is growing appropriately." d. "The test will help to determine if you have a boy or girl."

ANS: b Feedback a. Fetal position during pregnancy changes, and position in the first trimester is not indicative of position later in pregnancy. b. Fetal growth and size are fairly consistent during the first trimester and are a reliable indicator of the weeks of gestation. c. Fetal growth is best assessed later in pregnancy. d. The primary rationale for ultrasounds is not to determine gender.

Which of the following sites is priority for the nurse to assess when caring for a breastfeeding client, G8 P5, who is 1 hour postdelivery? a. Nipples b. Fundus c. Lungs d. Rectum

ANS: b Feedback a. Her nipples should be assessed, but this is not the priority assessment. b. This client is a grand multipara. She is high risk for uterine atony and postpartum hemorrhage. The nurse should monitor her fundus very carefully. c. Her lungs should be assessed bilaterally, but this is not the priority assessment. d. Her rectum should be assessed for hemorrhoids, but this is not the priority assessment.

Which of the following neonatal signs or symptoms would the nurse expect to see in a neonate with an elevated bilirubin level? a. Low glucose b. Poor feeding c. Hyperactivity d. Hyperthermia

ANS: b Feedback a. Hypoglycemia is not a sign that is related to an elevated bilirubin level. b. The baby is likely to feed poorly. An elevated bilirubin level adversely affects the central nervous system. Babies are often sleepy and feed poorly when the bilirubin level is elevated. c. Hyperactivity is the opposite of the behavior one would expect the baby to exhibit. d. Hyperthermia is not directly related to an elevated bilirubin level.

29. A labor nurse is caring for a patient, 39 weeks' gestation, who has been diagnosed with placenta previa. Which of the following physician orders should the nurse question? a. Type and cross-match her blood. b. Insert an internal fetal monitor electrode. c. Administer an oral stool softener. d. Assess her complete blood count.

ANS: b Feedback a. It would be appropriate to type and cross-match the patient for a blood transfusion. b. This action is inappropriate. When a patient has a placenta previa, nothing should be inserted into the vagina. c. To prevent constipation, it is appropriate for a patient to take a stool softener. d. It is appropriate to monitor the patient for signs of anemia.

27. A 34-weeks' gestation multigravida, G3 P1 is admitted to the labor suite. She is contracting every 7 minutes and 40 seconds. The woman has several medical problems. Which of the following of her comorbidities is most consistent with the clinical picture? a. Kyphosis b. Urinary tract infection c. Congestive heart failure d. Cerebral palsy

ANS: b Feedback a. Kyphosis is unrelated to preterm labor. b. Urinary tract infections often precipitate preterm labor. c. It is unlikely that the congestive heart failure precipitated the preterm labor. d. Cerebral palsy is unrelated to preterm labor.

5. During pregnancy, poorly controlled asthma can place the fetus at risk for: a. Hyperglycemia b. IUGR c. Hypoglycemia d. Macrosomia

ANS: b Feedback a. Maternal asthma does not place the fetus at risk for hyperglycemia. b. Compromised pulmonary function can lead to decompensation and hypoxia that decrease oxygen flow to the fetus and can cause intrauterine growth restriction (IUGR). c. Asthma does not directly affect glycemic control. d. A fetus experiencing hypoxia would be small for gestational age, not large for gestational age.

8. Which of the following medications administered to the pregnant client with GDM and experiencing preterm labor requires close monitoring of the client's blood glucose levels? a. Nifedipine b. Betamethasone c. Magnesium sulfate d. Indomethacin

ANS: b Feedback a. Nifedipine does not affect maternal blood glucose levels. b. Beta-sympathomimetics may stimulate hyperglycemia which will require an increased need for insulin. c. Magnesium sulfate does not affect blood glucose levels. d. Indomethacin does not affect blood glucose levels.

A full-term neonate who is 30 hours old has a bilirubin level of 10 mg/dL. The neonate has a yellowish tint to the skin of the face. The mother is breastfeeding her newborn. The nurse caring for this neonate would anticipate which of the following interventions? a. Phototherapy b. Feeding neonate every 2 to 3 hours c. Switch from breastfeeding to bottle feeding d. Assess red blood cell count

ANS: b Feedback a. Phototherapy is considered when the levels are 12 mg/dL or higher when the neonate is 25 to 48 hours old. Neonates re-absorb increased amounts of unconjugated bilirubin in the intestines due to lack of intestinal bacteria and decreased gastrointestinal motility. b. Adequate hydration promotes excretion of bilirubin in the urine. c. Colostrum acts as a laxative and assists in the passage of meconium. d. Assessing RBC is not a treatment for hyperbilirubinemia.

7. Which of the following laboratory values is most concerning in a client with pregnancy-induced hypertension? a. Total urine protein of 200 mg/dL b. Total platelet count of 40,000 mm c. Uric acid level of 8 mg/dL d. Blood urea nitrogen 24 mg/dL

ANS: b Feedback a. The client's urine protein is elevated. A urine protein of ≥300 mg/dL in a 24-hour collection is considered concerning. b. Correct. A platelet count of £50,000 is a critical value and should be reported to the health-care provider immediately. This client is at increased risk of hemorrhage. c. The uric acid level is only slightly elevated. d. The BUN is only slightly elevated.

23. A woman at 10 weeks' gestation is diagnosed with gestational trophoblastic disease (hydatiform mole). Which of the following findings would the nurse expect to see? a. Platelet count of 550,000/ mm3 b. Dark brown vaginal bleeding c. White blood cell count 17,000/ mm3 d. Macular papular rash

ANS: b Feedback a. The nurse would not expect to see an elevated platelet count. b. The nurse would expect to see dark brown vaginal discharge c. The nurse would not expect to see an elevated white blood cell count. d. The nurse would not expect to see a rash.

Your patient is a 28-year-old gravida 2 para 1 in active labor. She has been in labor for 12 hours. Upon further assessment, the nurse determines that she is experiencing a hypotonic labor pattern. Possible maternal and fetal implications from hypotonic labor patterns are: a. Intrauterine infection and maternal exhaustion with fetal distress usually occurring early in labor. b. Intrauterine infection and maternal exhaustion with fetal distress usually occurring late in labor. c. Intrauterine infection and postpartum hemorrhage with fetal distress early in labor. d. Intrauterine infection and ruptured uterus and fetal death.

ANS: b Feedback a. The risk of hypotonic labor occurs later in labor. b. Hypotonic labor patterns increase risk for infection and maternal exhaustion, with fetal distress occurring late in labor as hypotonic patterns prolong labor. c. There is not an increased risk of postpartum hemorrhage or fetal distress in early labor. d. Hypotonic patterns do not result in rupture of the uterus.

A primigravida woman at 42 weeks' gestation received Prepidil (dinoprostone) for induction 12 hours ago. The Bishop score is now 3. Which of the following actions by the nurse is appropriate? a. Perform Nitrazine analysis of the amniotic fluid. b. Report the lack of progress to the obstetrician. c. Place the woman on her left side. d. Ask the doctor for an order for oxytocin.

ANS: b Feedback a. There is nothing in the scenario that implies that the membranes may have ruptured. b. Little progress has taken place. The Bishop score of a primigravida will need to be 9 or higher before oxytocin will be effective. c. There is nothing in the scenario that implies that the patient needs to be placed on her side. d. The Bishop score of a primigravida will need to be 9 or higher before oxytocin will be effective.

Your pregnant patient is having maternal alpha-fetoprotein (AFP) screening. She does not understand how a test on her blood can indicate a birth defect in the fetus. The best reply by the nurse is :a. "We have done this test for a long time." b. "If babies have a neural tube defect, alpha-fetoprotein leaks out of the fetus and is absorbed into your blood, causing your level to rise. This serum blood test detects that rise." c. "Neural tube defects are a genetic anomaly, and we examine the amount of alpha-fetoprotein in your DNA." d. "If babies have a neural tube defect, this results in a decrease in your level of alpha-fetoprotein."

ANS: b Feedback a. This response does not explain AFP screening. b. When a neural tube defect is present, AFP is absorbed in the maternal circulation, resulting in a rise in the maternal AFP level. c. AFP testing is not related to DNA. d. Fetal neural tube defects result in an increase in maternal AFP.

During a cesarean section, which action by the nurse is done to prevent compression of the descending aorta and vena cava? a. Right lateral tilt b. Left lateral tilt c. Elevate head of gurney at 30 degrees d. Administration of IV fluid preload of 500 to 1000 mL

ANS: b Positioning of the patient with a left tilt maintains a left uterine displacement to decrease the risk of aortocaval compression related to compression on the aorta and inferior vena cava due to weight of the gravid uterus.

28. A baby has just been admitted into the neonatal intensive care unit with a diagnosis of intrauterine growth restriction (IUGR). Which of the following maternal problems could have resulted in this complication? (Select all that apply.) a. Cholecystitis b. Hypertension c. Cigarette smoker d. Candidiasis e. Cerebral palsy

ANS: b, c Babies born to women with cholecystitis are not especially high risk for IUGR. Babies born to women with PIH or who smoke are high risk for IUGR. Babies born to women with candidiasis or cerebral palsy are not especially high risk for IUGR.

A baby has just been admitted into the neonatal intensive care unit with a diagnosis of intrauterine growth restriction (IUGR). Which of the following maternal problems could have resulted in this complication? (Select all that apply.) a. Cholecystitis b. Hypertension c. Cigarette smoker d. Candidiasis e. Cerebral palsy

ANS: b, c Babies born to women with cholecystitis are not especially high risk for IUGR. Babies born to women with PIH or who smoke are high risk for IUGR. Babies born to women with candidiasis or cerebral palsy are not especially high risk for IUGR.

27. The perinatal nurse caring for Emily, a 24-year-old mother of an infant born at 26 weeks' gestation, is providing discharge teaching. Emily is going to travel to the specialty center approximately 200 miles away where her daughter is receiving care. The nurse tells Emily that it is normal for Emily to feel (select all that apply): a. In control b. Anxious c. Guilty d. Overwhelmed

ANS: b, c, d a. Parents usually feel out of control. b. Correct answer. c. Correct answer. d. Correct answer.

The perinatal nurse caring for Emily, a 24-year-old mother of an infant born at 26 weeks' gestation, is providing discharge teaching. Emily is going to travel to the specialty center approximately 200 miles away where her daughter is receiving care. The nurse tells Emily that it is normal for Emily to feel (select all that apply): a. In control b. Anxious c. Guilty d. Overwhelmed

ANS: b, c, d Feedback a. Parents usually feel out of control. b. Correct answer. c. Correct answer. d. Correct answer.

The nurse is caring for a recently immigrated Chinese woman in the postpartum unit. Based on cultural beliefs and practices of the woman, the nurse would anticipate which of the following? (Select all that apply.) a. The woman prefers cold water for drinking. b. The woman prefers not to shower. c. The woman prefers to have her female relatives care for her baby. d. The woman prefers to have her family bring her food to eat.

ANS: b, c, d In traditional Chinese beliefs and practices, the woman is to rest and female family members take care of the woman and her infant. During the first month, the woman is to avoid yin energy by eating specific foods and avoiding drinking or touching cold water.

11. A 1-day-old neonate in the well-baby nursery is suspected of suffering from drug withdrawal because he is markedly hyperreflexic and is exhibiting which of the following additional sign or symptom? a. Prolonged periods of sleep b. Hypovolemic anemia c. Repeated bouts of diarrhea d. Pronounced pustular rash

ANS: c a. Babies who are withdrawing from drugs have disorganized behavioral states and sleep very poorly. b. There is nothing in the scenario that indicates that this child is hypovolemic or anemic. c. Babies who are experiencing withdrawal often experience bouts of diarrhea. d. A pustular rash is characteristic of an infectious problem, not of neonatal abstinence syndrome.

4. A NICU nurse is caring for a full-term neonate being treated for group B streptococcus. The mother of the neonate is crying and shares that she cannot understand how her baby became infected. The best response by the nurse is: a. "Newborns are more susceptible to infections due to an immature immune system. Would you like additional information on the newborn immune system?" b. "The infection was transmitted to your baby during the birthing process. Do you have a history of sexual transmitted infections?" c. "Approximately 10% to 30% of women are asymptomatic carries of group B streptococcus which is found in the vaginal area. What other questions do you have regarding your baby's health?" d. "I see that this is very upsetting for you. I will come back later and answer your questions."

ANS: c a. Correct information, but does not fully address the woman's concern. b. Correct, but GBS is not a sexually transmitted disease. c. Correct. This response answers her questions and allows her to ask additional questions about her baby's health. d. Acknowledges that she is upset but does not provide immediate information.

7. A multipara, 26 weeks' gestation and accompanied by her husband, has just delivered a fetal demise. Which of the following nursing actions is appropriate at this time? a. Encourage the parents to pray for the baby's soul. b. Advise the parents that it is better for the baby to have died than to have had to live with a defect. c. Encourage the parents to hold the baby. d. Advise the parents to refrain from discussing the baby's death with their other children.

ANS: c a. It is inappropriate for the nurse to advise prayer. The parents must decide for themselves how they wish to express their spirituality. b. This is an inappropriate suggestion. c. This is an appropriate suggestion. Encouraging parents to spend time with their baby and hold their baby is an action that supports the parents during the grieving process. d. This is an inappropriate suggestion. It is very important for the parents to clearly communicate the baby's death with their other children.

8. The nurse is assessing a baby girl on admission to the newborn nursery. Which of the following findings should the nurse report to the neonatologist? a. Intermittent strabismus b. Startling c. Grunting d. Vaginal bleeding

ANS: c a. Pseudostrabismus is a normal finding. b. Startling is a normal finding. c. Grunting is a sign of respiratory distress. The neonatologist should be notified. d. Vaginal bleeding is a normal finding.

9. It is noted that the amniotic fluid of a 42-week gestation baby, born 30 seconds ago, is thick and green. Which of the following actions by the nurse is critical at this time? a. Perform a gavage feeding immediately. b. Assess the brachial pulse. c. Assist a physician with intubation. d. Stimulate the baby to cry.

ANS: c a. This action is not appropriate. The baby needs tracheal suctioning. b. The baby needs to have tracheal suctioning. The most important action to promote health for the baby is for the health-care team to establish an airway that is free of meconium. c. This action is appropriate. The baby needs to be intubated in order for deep suctioning to be performed by the physician. A nurse would not intubate and suction but rather would assist with the procedures. d. It is strictly contraindicated to stimulate the baby to cry until the trachea has been suctioned. The baby would aspirate the meconium-stained fluid, which could result in meconium-aspiration syndrome.

21. A woman at 32 weeks' gestation is diagnosed with severe preeclampsia with HELLP syndrome. The nurse will identify which of the following as a positive patient care outcome? a. Rise in serum creatinine b. Drop in serum protein c. Resolution of thrombocytopenia d. Resolution of polycythemia

ANS: c Feedback a. A rise in serum creatinine indicates that the kidneys are not effectively excreting creatinine. It is a negative outcome. b. A drop in serum protein indicates that the kidneys are allowing protein to be excreted. This is a negative outcome. c. Resolution of thrombocytopenia is a positive sign. It indicates that the platelet count is returning to normal. d. Polycythemia is not related to HELLP syndrome. Rather one sees a drop in red cell and platelet counts with HELLP. A positive sign, therefore, would be a rise in the RBC count.

A 1-day-old neonate in the well-baby nursery is suspected of suffering from drug withdrawal because he is markedly hyperreflexic and is exhibiting which of the following additional sign or symptom? a. Prolonged periods of sleep b. Hypovolemic anemia c. Repeated bouts of diarrhea d. Pronounced pustular rash

ANS: c Feedback a. Babies who are withdrawing from drugs have disorganized behavioral states and sleep very poorly. b. There is nothing in the scenario that indicates that this child is hypovolemic or anemic. c. Babies who are experiencing withdrawal often experience bouts of diarrhea. d. A pustular rash is characteristic of an infectious problem, not of neonatal abstinence syndrome.

A woman on the day of discharge from the postpartum unit requests clean towels so she can take a shower, asks a number of questions regarding breastfeeding, and shares that she is nervous about taking her baby home and not being able to remember everything she has been taught. These are behaviors associated with: a. Bonding b. Taking in c. Taking hold d. Attachment

ANS: c Feedback a. Bonding is defined as the emotional feelings that begin during pregnancy or shortly after birth between the parent and the newborn. Bonding is unidirectional from parent to newborn. b. In the taking-in phase, women are dependent and need assistance with self-care and care of the infant. c. Correct. These are common behaviors of women in the taking-hold phase. Women during this phase have moved to being more independent and able to initiate self-care. They are highly interested in learning about the care of their baby but can easily become frustrated and discouraged when they do not immediately master a new skill. d. Attachment is a connection that forms from parent to infant and infant to parent. Attachment has a lifelong impact on the developing individual.

33. A woman who is 36 weeks pregnant presents to the labor and delivery unit with a history of congestive heart disease. Which of the following findings should the nurse report to the primary health-care practitioner? a. Presence of chloasma b. Presence of severe heartburn c. 10-pound weight gain in a month d. Patellar reflexes +1

ANS: c Feedback a. Chloasma is a normal pregnancy finding. b. Heartburn is an expected finding during the third trimester. c. The weight gain may be due to fluid retention. Fluid retention may occur in patients with pregnancy-induced hypertension and in patients with congestive heart failure. The physician should be notified. d. Although slightly hyporeflexic, patellar reflexes of +1 are within normal limits.

A NICU nurse is caring for a full-term neonate being treated for group B streptococcus. The mother of the neonate is crying and shares that she cannot understand how her baby became infected. The best response by the nurse is: a. "Newborns are more susceptible to infections due to an immature immune system. Would you like additional information on the newborn immune system?" b. "The infection was transmitted to your baby during the birthing process. Do you have a history of sexual transmitted infections?" c. "Approximately 10% to 30% of women are asymptomatic carries of group B streptococcus which is found in the vaginal area. What other questions do you have regarding your baby's health?" d. "I see that this is very upsetting for you. I will come back later and answer your questions."

ANS: c Feedback a. Correct information, but does not fully address the woman's concern. b. Correct, but GBS is not a sexually transmitted disease. c. Correct. This response answers her questions and allows her to ask additional questions about her baby's health. d. Acknowledges that she is upset but does not provide immediate information.

A nurse is preparing a woman in early labor for an urgent cesarean birth related to breech presentation. Select the best nursing action for reducing the couple's anxiety levels. a. Explain the reason for the need for a cesarean section .b. Inform parents that their baby is in distress. c. Ask the couple to share their concerns. d. Reassure the couple that both the woman and baby are in no danger.

ANS: c Feedback a. Explaining the reason she is having a cesarean birth is helpful but may not address their concerns. b. It is important to acknowledge that the baby is stable, but this response does not allow the couple to share their concerns that may be causing an increase in anxiety. c. By asking the couple to share their concerns, the nurse can address these concerns. d. Reassuring the couple that the woman and baby are in no danger is correct, but it is not the best answer because it does not allow the couple to verbalize their concerns.

The perinatal nurse is providing care to Carol, a 28-year-old multiparous woman in labor. Upon arrival to the birthing suite, Carol was 7 cm dilated and experiencing contractions every 1 to 2 minutes which she describes as "strong." Carol states she labored for 1 hour at home. As the nurse assists Carol from the assessment area to her labor and birth room, Carol states that she is feeling some rectal pressure. Carol is most likely experiencing: a. Hypertonic contractions b. Hypotonic contractions c. Precipitous labor d. Uterine hyperstimulation

ANS: c Feedback a. Hypertonic contractions result in little cervical change. b. Hypotonic contractions result in little cervical change. c. Contrary to both hypertonic and hypotonic labor, precipitate labor contractions produce very rapid, intense contractions. A precipitous labor lasts less than 3 hours from the beginning of contractions to birth. Patients often progress through the first stage of labor with little or no pain and may present to the birth setting already advanced into the second stage of labor. d. Patients with precipitous labor often progress through the first stage of labor with little or no pain and may present to the birth setting already advanced into the second stage of labor. Precipitous labor contractions produce very rapid, intense contractions.

During labor induction with oxytocin, the fetal heart rate baseline is in the 140s with moderate variability. Contraction frequency is assessed to be every 2 minutes with duration of 60 seconds, of moderate strength to palpation. Based on this assessment, the nurse should take which action? a. Increase oxytocin infusion rate per physician's protocol. b. Stop oxytocin infusion immediately. c. Maintain present oxytocin infusion rate and continue to assess. d. Decrease oxytocin infusion rate by 2 mU/min and report to physician.

ANS: c Feedback a. Increasing the oxytocin infusion could result in uterine hyperstimulation. b. The uterine contraction pattern is normal, and oxytocin infusion should be maintained, not stopped. c. Correct. Maintain present oxytocin infusion rate and continue to assess is the correct response, as this question describes a normal uterine contraction pattern. d. The uterine contraction pattern is normal, and oxytocin infusion should be maintained, not stopped or decreased.

2. A pregnant client with a history of multiple sexual partners is at highest risk for which of the following complications: a. Premature rupture of membranes b. Gestational diabetes c. Ectopic pregnancy d. Pregnancy-induced hypertension

ANS: c Feedback a. Multiple partners do not increase a woman's risk of premature rupture of membranes. b. Genetics and client diet and weight are contributing factors to gestational diabetes. c. Correct. A history of multiple sexual partners places the client at a higher risk of having contracted a sexually transmitted disease that could have ascended the uterus to the fallopian tubes and caused fallopian tube blockage, placing the client at high risk for an ectopic pregnancy. d. Multiple sexual partners are not a risk factor for pregnancy-induced hypertension.

A woman who is 12 weeks postpartum presents with the following behavior: she reports severe mood swings and hearing voices, believes her infant is going to die, she has to be reminded to shower and put on clean clothes, and she feels she is unable to care for her baby. These behaviors are associated with which of the following? a. Postpartum blues b. Postpartum depression c. Postpartum psychosis d. Maladaptive mother-infant attachment

ANS: c Feedback a. Postpartum blues usually occurs within the first few weeks of the postpartum period. Women experiencing postpartum blues will have mild mood swings, and they can take care of themselves as well as their baby. b. Women with PPD are predominately depressed and do not have mood swings. c. Postpartum psychosis is associated with a break from reality reflected in the woman hearing voices. d. The symptoms reported are reflective of a psychiatric disorder beyond maladaptive attachment.

The nurse is assessing a baby girl on admission to the newborn nursery. Which of the following findings should the nurse report to the neonatologist? a. Intermittent strabismus b. Startling c. Grunting d. Vaginal bleeding

ANS: c Feedback a. Pseudostrabismus is a normal finding. b. Startling is a normal finding. c. Grunting is a sign of respiratory distress. The neonatologist should be notified. d. Vaginal bleeding is a normal finding.

16. A woman who has had no prenatal care was assessed and found to have hydramnios on admission to the labor unit and has since delivered a baby weighing 4500 grams. Which of the following complications of pregnancy likely contributed to these findings? a. Pyelonephritis b. Pregnancy-induced hypertension c. Gestational diabetes d. Abruptio placentae

ANS: c Feedback a. Pyelonephritis does not lead to the development of hydramnios or macrosomia. b. Pregnancy-induced hypertension does not lead to the development of hydramnios or macrosomia. c. Untreated gestational diabetics often have hydramnios and often deliver macrosomic babies. d. Abruptio placentae does not lead to the development of hydramnios or macrosomia.

13. A patient is receiving magnesium sulfate for severe preeclampsia. The nurse must notify the attending physician immediately of which of the following findings? a. Patellar and biceps reflexes of +4 b. Urinary output of 50 mL/hr c. Respiratory rate of 10 rpm d. Serum magnesium level of 5 mg/dL

ANS: c Feedback a. The magnesium sulfate has been ordered because the patient has severe pregnancy-induced hypertension. Patellar and biceps reflexes of +4 are symptoms of the disease. b. The urinary output must be above 25 mL/hr. c. The drop in respiratory rate may indicate that the patient is suffering from magnesium toxicity. The nurse should report the finding to the physician. d. The therapeutic range of magnesium is 4 to 7 mg/dL.

A postpartum nurse has received an exchange report on the four following mother-baby couplets. Based on the provided information, which couplet should the nurse first assess? a. A 25-year-old G2P1 woman who is 36 hours postbirth and is having difficulty breastfeeding her baby girl. Her fundus is firm at the umbilicus, and lochia is moderate to scant. b. A 16-year-old G1P0 who will be discharged in the afternoon. It was reported that she refers to her baby boy as "it" and that she requested to have her baby stay in the nursery so she could sleep. c. A 32-year-old G5P4 woman who delivered a 4500 gram baby boy 2 hours ago after a 20 hour labor that was augmented. It was reported that her fundus is 2 cm above umbilicus with moderate lochia. d. A 28-year-old G2P1 woman who delivered a 3800 gram baby girl by elective cesarean birth. She had spinal anesthesia and was given intrathecal preservative-free morphine for postoperative pain management. Her vital signs are B/P 115/75, P 80, R 18 T 98.

ANS: c Feedback a. The priority need for this woman is breastfeeding assistance which does not require immediate attention. b. The data indicate that the woman is experiencing a delay in bonding and that social services should become involved. This needs to be done prior to discharge but does not require immediate attention. c. This woman is at risk for hemorrhage (large baby, prolonged labor, augmented labor, high parity, and immediate postpartum). This woman needs to be assessed first to determine whether the fundus is firm and if lochia is within normal limits. d. Based on data provided, this woman is stable, but should be assessed second.

The NICU nurse recognizes that respiratory distress syndrome results from a developmental lack of: a. Lecithin b. Calcium c. Surfactant d. Magnesium

ANS: c Feedback a. The ratio of lecithin to sphingomyelin in the amniotic fluid is used to assess maturity of fetal lungs. b. Calcium is needed to prevent undermineralization of bones. c. Respiratory distress syndrome (RDS) is a developmental respiratory disorder that affects preterm newborns due to lack of lung surfactant. The pathology of RDS is that there is diffuse atelectasis with congestion and edema in the lung spaces. On deflation, the alveoli collapse, and there is decreased lung compliance. d. Magnesium is needed to prevent undermineralization of bones.

25. A woman who is admitted to labor and delivery at 30 weeks' gestation, is 1 cm dilated, and is contracting q 5 minutes. She is receiving magnesium sulfate IV piggyback. Which of the following maternal vital signs is most important for the nurse to assess each hour? a. Temperature b. Pulse c. Respiratory rate d. Blood pressure

ANS: c Feedback a. The temperature should be monitored, but it is not the most important vital sign. b. The pulse rate should be monitored, but it is not the most important vital sign. c. The respiratory rate is the most important vital sign. Respiratory depression is a sign of magnesium toxicity. d. The blood pressure should be monitored, but it is not the most important vital sign.

It is noted that the amniotic fluid of a 42-week gestation baby, born 30 seconds ago, is thick and green. Which of the following actions by the nurse is critical at this time? a. Perform a gavage feeding immediately. b. Assess the brachial pulse. c. Assist a physician with intubation. d. Stimulate the baby to cry.

ANS: c Feedback a. This action is not appropriate. The baby needs tracheal suctioning. b. The baby needs to have tracheal suctioning. The most important action to promote health for the baby is for the health-care team to establish an airway that is free of meconium. c. This action is appropriate. The baby needs to be intubated in order for deep suctioning to be performed by the physician. A nurse would not intubate and suction but rather would assist with the procedures. d. It is strictly contraindicated to stimulate the baby to cry until the trachea has been suctioned. The baby would aspirate the meconium-stained fluid, which could result in meconium-aspiration syndrome.

24. After an education class, the nurse overhears an adolescent woman discussing safe sex practices. Which of the following comments by the young woman indicates that additional teaching about sexually transmitted infection (STI) control issues is needed? a. "I could get an STI even if I just have oral sex." b. "Girls over 16 are less likely to get STDs than younger girls." c. "The best way to prevent an STI is to use a diaphragm." d. "Girls get human immunodeficiency virus (HIV) easier than boys do."

ANS: c Feedback a. This statement is true. Organisms that cause sexually transmitted infections can invade the respiratory and gastrointestinal tracts. b. This statement is true. Young women are especially high risk for becoming infected with sexually transmitted diseases. c. This statement is untrue. The young woman needs further teaching. Condoms protect against STDs and pregnancy. In addition, condoms can be kept in readiness for whenever sex may occur spontaneously. Using condoms does not require the teen to plan to have sex. A diaphragm is not an effective infection-control method. Plus, it would require the teen to plan for intercourse. d. This statement is true. Young women are higher risk for becoming infected with HIV than are young men.

The perinatal nurse accurately defines postpartum hemorrhage by including a decrease in hematocrit levels from pre- to postbirth by: a. 5% b. 8% c. 10% d. 15%

ANS: c Historically, practitioners have defined postpartum hemorrhage as a blood loss greater than 500 mL following a vaginal birth and 1000 mL or more following a cesarean birth. Hematocrit levels that decrease 10% from pre- to postbirth measurements are also included in the definition.

The perinatal nurse notifies the physician of the findings related to Juanita's assessment. The first step in care will most likely be to: a. Prepare Juanita for surgery b. Administer intravenous fluids c. Apply ice to the perineum d. Insert a urinary catheter

ANS: c If the hematoma is less than 3 to 5 centimeters in diameter, the physician usually orders palliative treatments such as ice to the area for the first 12 hours along with pain medication. After 12 hours, sitz baths are prescribed to replace the application of ice. However, a hematoma larger than 5 centimeters may require incision and drainage with the possible placement of a drain.

26. You are caring for a patient who was admitted to labor and delivery at 32 weeks' gestation and diagnosed with preterm labor. She is currently on magnesium sulfate, 2 gm per hour. Upon your initial assessment you note that she has a respiratory rate of 8 with absent deep tendon reflexes. What will be your first nursing intervention? a. Elevate head of the bed b. Notify the MD c. Discontinue magnesium sulfate d. Draw a serum magnesium level

ANS: c Initial nursing intervention needs to be discontinuing magnesium sulfate because the patient is exhibiting signs of magnesium toxicity with absent deep tendon reflexes and decreased respiratory rate.

21. Nursing actions that minimize oxygen demands in the neonate include which of the following? (Select all that apply.) a. Providing frequent rest breaks when feeding b. Placing neonate on back for sleeping c. Maintaining a neutral thermal environment (NTE) d. Clustering nursing care

ANS: c, d a. A prolonged feeding session increases energy consumption that increases oxygen consumption. b. Placing the neonate on the back for sleeping has no effect on oxygen consumption. c. A decrease in environmental temperature leads to a decrease in the neonate's body temperature which leads to an increase in respiratory and heart rate that leads to an increase in oxygen consumption. d. Clustering of nursing care decreases stress which decreases oxygen requirements.

Nursing actions that minimize oxygen demands in the neonate include which of the following? (Select all that apply.) a. Providing frequent rest breaks when feeding b. Placing neonate on back for sleeping c. Maintaining a neutral thermal environment (NTE) d. Clustering nursing care

ANS: c, d Feedback a. A prolonged feeding session increases energy consumption that increases oxygen consumption. b. Placing the neonate on the back for sleeping has no effect on oxygen consumption. c. A decrease in environmental temperature leads to a decrease in the neonate's body temperature which leads to an increase in respiratory and heart rate that leads to an increase in oxygen consumption. d. Clustering of nursing care decreases stress which decreases oxygen requirements.

45. The perinatal nurse knows that tocolytic agents are most often used to (select all that apply): a. Prevent maternal infection b. Prolong pregnancy to 40 weeks' gestation c. Prolong pregnancy to facilitate administration of antenatal corticosteroids d. Allow for transport of the woman to a tertiary care facility

ANS: c, d Feedback a. Tocolytics are not used to treat maternal infection. b. Tocolytics are generally only effective in delaying delivery for several days. c. Presently, it is believed that the best reason to use tocolytic drugs is to allow an opportunity to begin the administration of antenatal corticosteroids to accelerate fetal lung maturity. d. Delaying the birth provides time for maternal transport to a facility equipped with a neonatal intensive care unit.

Which of the following nursing actions are directed at assisting men in their transition to fatherhood? (Select all that apply.) a. Encourage the woman to take on the major responsibility for infant care. b. Talk to the man, away from his partner, about his expectations of the fathering role. c. Praise the father for his interactions with his infant. d. Provide information on infant care and behavior to both parents.

ANS: c, d It is important to first have the couple discuss with each other their expectations of the fathering role. Once this has occurred, then the woman and nurse need to support the man in his role of infant care. Both parents need to receive information about infant care and infant behaviors, and both parents need to be praised for their interactions with their baby.

26. A baby was born 4 days ago at 34 weeks' gestation. She is receiving phototherapy as ordered by the physician for physiological jaundice. She has symptoms of temperature instability, dry skin, poor feeding, lethargy, and irritability. The nurse's priority nursing action(s) is (are) to (select all that apply): a. Verify laboratory results to check for hypomagnesia. b. Verify laboratory results to check for hypoglycemia. c. Monitor the baby's temperature to check for hypothermia. d. Calculate 24-hour intake and output to check for dehydration

ANS: c, d There are two priority nursing interventions for hyperbilirubinemia. Hydration status is important if the newborn shows signs of dehydration such as dry skin and mucus membranes, poor intake, concentrated urine or limited urine output, and irritability. The newborn should also be kept warm while receiving phototherapy. When an infant is under phototherapy, the temperature needs to be monitored closely because the lights give off extra heat, but if the newborn is in an open crib and undressed, hypothermia may occur. Hypomagnesia and hypoglycemia are not related to phototherapy.

A baby was born 4 days ago at 34 weeks' gestation. She is receiving phototherapy as ordered by the physician for physiological jaundice. She has symptoms of temperature instability, dry skin, poor feeding, lethargy, and irritability. The nurse's priority nursing action(s) is (are) to (select all that apply): a. Verify laboratory results to check for hypomagnesia. b. Verify laboratory results to check for hypoglycemia. c. Monitor the baby's temperature to check for hypothermia. d. Calculate 24-hour intake and output to check for dehydration.

ANS: c, d There are two priority nursing interventions for hyperbilirubinemia. Hydration status is important if the newborn shows signs of dehydration such as dry skin and mucus membranes, poor intake, concentrated urine or limited urine output, and irritability. The newborn should also be kept warm while receiving phototherapy. When an infant is under phototherapy, the temperature needs to be monitored closely because the lights give off extra heat, but if the newborn is in an open crib and undressed, hypothermia may occur. Hypomagnesia and hypoglycemia are not related to phototherapy.

Which of the following is a medical indication for a cesarean birth? (Select all that apply.) a. Maternal blood pressure of 130/90 b. Cervical dilation of 1.5 cm per hour during the active phase of labor c. Late deceleration of the fetal heart rate with minimal variability d. Complete placenta previae. Arrest of fetal descent

ANS: c, d, eA maternal blood pressure of 130/90 may be an indication of mild PHI which is not a medical indication for cesarean birth. Cervical dilation of 1.5 cm/minutes is within normal limits for cervical changes during the active phase. Late decelerations combined with minimal variability in the fetal heart rate reflect fetal intolerance of labor and are an indication for cesarean birth. A complete placenta previa covers the internal os necessitating a cesarean birth. Arrest of fetal descent indicates cephalopelvic disproportion.

A woman is 3 hours post-early-postpartum hemorrhage of 800 mL at delivery. Select the nursing actions for care of this patient. (Select all that apply.) a. Limit fluid intake to prevent nausea and vomiting. b. Assess fundus every 4 hours during the first 8 hours. c. Explain the importance of preventing an overdistended bladder. d. Provide assistance with ambulation.

ANS: c, dFluid intake should be increased following a postpartum hemorrhage to decrease the risk of hypovolemia. The fundus should be assessed a minimum of every hour for the first 4 hours following a PPH. The woman needs to know the importance of preventing an overdistended bladder to decrease the risk of further hemorrhage. After postpartum hemorrhage, a woman is at risk for orthostatic hypotension.

29. The perinatal nurse assessing a newborn for jaundice recalls that __________ is a process that converts the yellow lipid-soluble (nonexcretable) bilirubin pigment (present in bile) into a water-soluble (excretable) pigment.

ANS: conjugation Conjugation of bilirubin constitutes a major function of the newborn's liver. Conjugation is a process that converts the yellow lipid-soluble (nonexcretable) bilirubin pigment (present in bile) into a water-soluble (excretable) pigment.

The perinatal nurse assessing a newborn for jaundice recalls that __________ is a process that converts the yellow lipid-soluble (nonexcretable) bilirubin pigment (present in bile) into a water-soluble (excretable) pigment.

ANS: conjugation Conjugation of bilirubin constitutes a major function of the newborn's liver. Conjugation is a process that converts the yellow lipid-soluble (nonexcretable) bilirubin pigment (present in bile) into a water-soluble (excretable) pigment.

6. The following four babies are in the neonatal nursery. Which of the babies should be seen by the neonatologist as soon as possible? a. 1-day-old, HR 170 bpm, crying b. 2-day-old, T 98.9°F, slightly jaundice c. 3-day-old, breastfeeding q 2 h, rooting d. 4-day-old, RR 70 rpm, dusky coloring

ANS: d a. A slight tachycardia—170 bpm—is normal when a baby is crying. b. Slight jaundice on day 2 is within normal limits. c. It is normal for a breastfed baby to feed every 2 hours. d. A dusky skin color is abnormal in any neonate, whether or not the respiration rate is normal, although this baby is also slightly tachypneic.

2. A nurse assesses that a 3-day-old neonate who was born at 34 weeks' gestation has abdominal distention and vomiting. These assessment findings are most likely related to: a. Respiratory Distress Syndrome (RDS) b. Bronchopulmonary Dysplasia (BPD) c. Periventricular Hemorrhage (PVH) d. Necrotizing Enterocolitis (NEC)

ANS: d a. Assessment findings for RDS include tachypnea, intercostal retractions, respiratory grunting, and nasal flaring. b. Assessment findings for BPD include chest retractions; audible wheezing, rales, and rhonchi; hypoxia; and bronchospasm. c. Assessment findings for PVH include bradycardia, hypotonia, full and/or tense anterior fontanel, and hyperglycemia. d. Assessment findings related to NEC include abdominal distention, bloody stools, abdominal distention, vomiting, and increased gastric residual. These signs and symptoms are related to the premature neonate's inability to fully digest stomach contents and limitation in absorptive function.

5. A nursery nurse observes that a full-term AGA neonate has nasal congestion, hypertonia, and tremors and is extremely irritable. Based on these observations, the nurse suspects which of the following? a. Hypoglycemia b. Hypercalcemia c. Cold stress d. Neonatal withdrawal

ANS: d a. Signs and symptoms of hypoglycemia are jitteriness, hypotonia, irritability, apnea, lethargy, and temperature instability, but not nasal congestion. b. Signs and symptoms of hypercalcemia are vomiting, constipation, and cardiac arrhythmias. c. Signs and symptoms of cold stress are decreased temperature, cool skin, lethargy, pallor, tachypnea, hypotonia, jitteriness, weak cry, and grunting. d. These are common signs and symptoms of neonatal withdrawal.

The perinatal nurse listens as Chantal describes her labor and emergency cesarean birth. Providing an opportunity to review this experience may assist Chantal in: a. Her role development in the "letting go" stage b. Decreasing her ambivalence about her labor and birth c. Understanding her guilt involved in her labor and birth d. Developing more positive feelings about her labor and birth

ANS: d After a cesarean birth, especially when unplanned, nurses must be aware of the myriad of potential psychological issues that may arise. Research suggests that women may perceive cesarean birth to be a less positive experience than a vaginal birth. Unplanned or emergent cesarean deliveries and the experience of cesarean birth may be associated with more negative perceptions of the birthing experience. Allowing Chantal to talk about the experience can help her develop a more positive attitude about her own experience.

A patient, G1 P0, is admitted to the labor and delivery unit for induction of labor. The following assessments were made on admission: Bishop score of 4, fetal heart rate 140s with good variability and no decelerations, TPR 98.6ºF, 88, 20, BP 120/80, negative obstetrical history. A prostaglandin suppository was inserted at that time. Which of the following findings, 6 hours after insertion, would warrant the removal of the Cervidil (dinoprostone)? a. Bishop score of 5 b. Fetal heart of 152 bpm c. Respiratory rate of 24 rpm d. Contraction frequency of every 2 minutes

ANS: d Feedback a. A Bishop score of 9 or higher indicates that the primigravida woman's cervix is ripe. b. A fetal heart rate of 152 is within normal limits for this fetus. c. A respiratory rate of 24 is within normal limits. d. Cervidil should be removed for tachysystole.

The following four babies are in the neonatal nursery. Which of the babies should be seen by the neonatologist as soon as possible? a. 1-day-old, HR 170 bpm, crying b. 2-day-old, T 98.9°F, slightly jaundice c. 3-day-old, breastfeeding q 2 h, rooting d. 4-day-old, RR 70 rpm, dusky coloring

ANS: d Feedback a. A slight tachycardia—170 bpm—is normal when a baby is crying. b. Slight jaundice on day 2 is within normal limits. c. It is normal for a breastfed baby to feed every 2 hours. d. A dusky skin color is abnormal in any neonate, whether or not the respiration rate is normal, although this baby is also slightly tachypneic.

A client is 1 hour postpartum from a vacuum delivery over a midline episiotomy of a 4500-gram neonate. Which of the following nursing diagnoses is appropriate for this mother? a. Risk for altered parenting b. Risk for imbalanced nutrition: less than body requirements c. Risk for ineffective individual coping d. Risk for fluid volume deficit

ANS: d Feedback a. Although the baby is macrosomic, there is no evidence that this mother is high risk for altered parenting. b. This woman's baby is macrosomic—there is no indication that this woman is consuming a diet that is less than body requirements. c. There is no evidence that this mother is high risk for altered coping. d. This client is high risk for fluid volume deficit. Women who deliver macrosomic babies are high risk for uterine atony, which can lead to heavy flow of lochia.

The primary complications of amniocentesis are: a. Damage to fetal organs b. Puncture of umbilical cord c. Maternal pain d. Infection

ANS: d Feedback a. Amniocentesis is done under ultrasound guidance, and damage to fetal organs is very rare. b. Amniocentesis is done under ultrasound guidance, and damage to the umbilical cord is very rare. c. Amniocentesis is done under local anesthesia, and maternal pain is generally minimal. d. Amniocentesis involves insertion of a needle into the amniotic sac, and infection is the primary complication.

Your patient is a 37-year-old pregnant woman who is 5 weeks pregnant and is considering genetic testing. During your discussion, the woman asks the nurse what the advantages of chorionic villus sampling (CVS) are over amniocentesis. The best response is: a. "You will need anesthesia for amniocentesis, but not for CVS." b. "CVS is a faster procedure." c. "CVS provides more detailed information than amniocentesis." d. "CVS can be done earlier in your pregnancy, and the results are available more quickly."

ANS: d Feedback a. Anesthesia is not done for either procedure. b. The length of time for either procedure is similar. c. Both amniocentesis and CVS provide the same information. d. CVS can be done earlier in gestation.

A nurse assesses that a 3-day-old neonate who was born at 34 weeks' gestation has abdominal distention and vomiting. These assessment findings are most likely related to: a. Respiratory Distress Syndrome (RDS) b. Bronchopulmonary Dysplasia (BPD) c. Periventricular Hemorrhage (PVH) d. Necrotizing Enterocolitis (NEC)

ANS: d Feedback a. Assessment findings for RDS include tachypnea, intercostal retractions, respiratory grunting, and nasal flaring. b. Assessment findings for BPD include chest retractions; audible wheezing, rales, and rhonchi; hypoxia; and bronchospasm. c. Assessment findings for PVH include bradycardia, hypotonia, full and/or tense anterior fontanel, and hyperglycemia. d. Assessment findings related to NEC include abdominal distention, bloody stools, abdominal distention, vomiting, and increased gastric residual. These signs and symptoms are related to the premature neonate's inability to fully digest stomach contents and limitation in absorptive function.

22. A 16-year-old patient is admitted to the hospital with a diagnosis of severe preeclampsia. The nurse must closely monitor the woman for which of the following? a. High leukocyte count b. Explosive diarrhea c. Fractured pelvis d. Low platelet count

ANS: d Feedback a. High leukocyte count is not associated with severe pregnancy-induced hypertension (PIH) or HELLP (hemolysis, elevated liver enzymes, and low platelets) syndrome. b. Explosive diarrhea is not associated with severe PIH or HELLP syndrome. c. A fractured pelvis is not associated with severe PIH or HELLP syndrome. d. Low platelet count is one of the signs associated with HELLP (hemolysis, elevated liver enzymes, and low platelets) syndrome.

A 37-year-old woman who is 17 weeks pregnant has had an amniocentesis. Before discharge, the nurse teaches the woman to call her doctor if she experiences which of the following side effects? a. Pain at the puncture site b. Macular rash on the abdomen c. Decrease in urinary output d. Cramping of the uterus

ANS: d Feedback a. It is normal for the patient to experience pain at the puncture site. b. A rash is not an expected complication. c. Oliguria is not an expected complication. d. The woman should report any uterine cramping. Although rare, amniocentesis could stimulate preterm labor.

1. A client on 2 gm/hr of magnesium sulfate has decreased deep tendon reflexes. Identify the priority nursing assessment to ensure client safety. a. Assess uterine contractions continuously. b. Assess fetal heart rate continuously. c. Assess urinary output. d. Assess respiratory rate.

ANS: d Feedback a. Monitoring contractions does not indicate magnesium toxicity. b. Magnesium sulfate will decrease fetal variability and not provide an accurate assessment of magnesium toxicity. c. Urinary output does not correlate to decreased deep tendon reflexes. d. Correct. Respiratory effort and deep tendon reflexes (DTRs) are involuntary, and a decrease in DTRs could indicate the risk of magnesium sulfate toxicity and the risk for decreased respiratory effort.

18. The nurse is caring for two laboring women. Which of the patients should be monitored most carefully for signs of placental abruption? a. The patient with placenta previa b. The patient whose vagina is colonized with group B streptococci c. The patient who is hepatitis B surface antigen positive d. The patient with eclampsia

ANS: d Feedback a. Patients with placenta previa are not especially high risk for placental abruption. b. Patients colonized with group B streptococci are not especially high risk for placental abruption. c. Patients who are hepatitis B surface antigen positive are not especially high risk for placental abruption. d. Patients with eclampsia are high risk for placental abruption.

12. The perinatal nurse knows that the term to describe a woman at 26 weeks' gestation with a history of elevated blood pressure who presents with a urine showing 2+ protein (by dipstick) is: a. Preeclampsia b. Chronic hypertension c. Gestational hypertension d. Chronic hypertension with superimposed preeclampsia

ANS: d Feedback a. Preeclampsia is a multisystem, vasopressive disease process that targets the cardiovascular, hematologic, hepatic, and renal and central nervous systems. b. Chronic hypertension is hypertension that is present and observable prior to pregnancy or hypertension that is diagnosed before the 20th week of gestation. c. Gestational hypertension is a nonspecific term used to describe the woman who has a blood pressure elevation detected for the first time during pregnancy, without proteinuria. d. The following criteria are necessary to establish a diagnosis of superimposed preeclampsia: hypertension and no proteinuria early in pregnancy (prior to 20 weeks' gestation) and new-onset proteinuria, a sudden increase in protein—urinary excretion of 0.3 g protein or more in a 24-hour specimen, or two dipstick test results of 2+ (100 mg/dL), with the values recorded at least 4 hours apart, with no evidence of urinary tract infection; a sudden increase in blood pressure in a woman whose blood pressure has been well controlled; thrombocytopenia (platelet count lower than 100,000/mmC); and an increase in the liver enzymes alanine transaminase (ALT) or aspartate transaminase (AST) to abnormal levels.

11. The perinatal nurse is assessing a woman in triage who is 34 + 3 weeks' gestation in her first pregnancy. She is worried about having her baby "too soon," and she is experiencing uterine contractions every 10 to 15 minutes. The fetal heart rate is 136 beats per minute. A vaginal examination performed by the health-care provider reveals that the cervix is closed, long, and posterior. The most likely diagnosis would be: a. Preterm labor b. Term labor c. Back labor d. Braxton-Hicks contractions

ANS: d Feedback a. Preterm labor (PTL) is defined as regular uterine contractions and cervical dilation before the end of the 36th week of gestation. Many patients present with preterm contractions, but only those who demonstrate changes in the cervix are diagnosed with preterm labor. b. Term labor occurs after 37 weeks' gestation. c. There is no indication in this scenario that this is back labor. d. Braxton-Hicks contractions are regular contractions occurring after the third month of pregnancy. They may be mistaken for regular labor, but unlike true labor, the contractions do not grow consistently longer, stronger, and closer together, and the cervix is not dilated. Some patients present with preterm contractions, but only those who demonstrate changes in the cervix are diagnosed with preterm labor.

Your patient is 34 weeks pregnant and during a regular prenatal visit tells you she does not understand how to do "kick counts." The best response by the nurse would be to explain: a. "Here is an information sheet on how to do kick counts." b. "It is not important to do kick counts because you have a low-risk pregnancy." c. "Fetal kick counts are not a reliable indicator of fetal well-being in the third trimester." d. "Fetal movements are an indicator of fetal well-being. You should count twice a day, and you should feel 10 fetal movements in 2 hours."

ANS: d Feedback a. Providing written information may not be enough, and the patient may need a verbal explanation. b. Kick counts are indicated for all pregnancies. c. Kick counts are a reliable indicator of fetal well-being after 32 to 34 weeks' gestation. d. This response provides the patient with information on how to do kick counts and the rationale for doing kick counts and criteria for normal fetal movement.

19. The nurse is caring for a woman at 28 weeks' gestation with a history of preterm delivery. Which of the following laboratory data should the nurse carefully assess in relation to this diagnosis? a. Human relaxin levels b. Amniotic fluid levels c. Alpha-fetoprotein levels d. Fetal fibronectin levels

ANS: d Feedback a. Relaxin levels are rarely assessed. In addition, they are unrelated to the incidence of preterm labor. b. Amniotic fluid levels are not directly related to the incidence of preterm labor. c. Alpha-fetoprotein levels are not related to the incidence of preterm labor. d. A rise in the fetal fibronectin levels in cervical secretions has been associated with preterm labor.

A nursery nurse observes that a full-term AGA neonate has nasal congestion, hypertonia, and tremors and is extremely irritable. Based on these observations, the nurse suspects which of the following? a. Hypoglycemia b. Hypercalcemia c. Cold stress d. Neonatal withdrawal

ANS: d Feedback a. Signs and symptoms of hypoglycemia are jitteriness, hypotonia, irritability, apnea, lethargy, and temperature instability, but not nasal congestion. b. Signs and symptoms of hypercalcemia are vomiting, constipation, and cardiac arrhythmias. c. Signs and symptoms of cold stress are decreased temperature, cool skin, lethargy, pallor, tachypnea, hypotonia, jitteriness, weak cry, and grunting. d. These are common signs and symptoms of neonatal withdrawal.

A client delivered a 2800-gram neonate 4 hours ago by cesarean section with epidural anesthesia. Which of the following interventions should the nurse perform on the mother at this time? a. Maintain the client flat in bed. b. Assess the client's patellar reflexes. c. Monitor hourly urinary outputs. d. Assess the client's respiratory rate.

ANS: d Feedback a. The client should be assisted to a position of comfort. b. There is no indication in the scenario that the client must have her reflexes assessed. c. The client's hydration should be monitored postsurgery, but hourly assessments are unnecessary. d. The client has undergone major abdominal surgery. Her respiratory function should be assessed regularly.

31. According to agency policy, the perinatal nurse provides the following intrapartal nursing care for the patient with preeclampsia: a. Take the patient's blood pressure every 6 hours b. Encourage the patient to rest on her back c. Notify the physician of a urine output greater than 30 mL/hr d. Administer magnesium sulfate according to agency policy

ANS: d Feedback a. The nurse is the manager of care for the woman with preeclampsia during the intrapartal period. Careful assessments are critical. The blood pressure is taken every 1 hour or more frequently according to physician orders or institutional protocol. b. The nurse is the manager of care for the woman with preeclampsia during the intrapartal period. Careful assessments are critical. The patient should be encouraged to assume a side-lying position to enhance uterine perfusion. c. The nurse is the manager of care for the woman with preeclampsia during the intrapartal period. Careful assessments are critical. A urine output less than 30 mL/hr is indicative of oliguria and the physician must be notified. d. The nurse is the manager of care for the woman with preeclampsia during the intrapartal period. Careful assessments are critical. The nurse administers medications as ordered and should adhere to hospital protocol for a magnesium sulfate infusion.

14. A woman in labor and delivery is being given subcutaneous terbutaline for preterm labor. Which of the following common medication effects would the nurse expect to see in the mother? a. Serum potassium level increases b. Diarrhea c. Urticaria d. Complaints of nervousness

ANS: d Feedback a. The nurse would not expect to see a rise in the mother's serum potassium levels. b. The beta agonists are not associated with diarrhea. c. The beta agonists are not associated with urticaria. d. Complaints of nervousness are commonly made by women receiving subcutaneous beta agonists.

30. A type 1 diabetic patient has repeatedly experienced elevated serum glucose levels throughout her pregnancy. Which of the following complications of pregnancy would the nurse expect to see? a. Postpartum hemorrhage b. Neonatal hyperglycemia c. Postpartum oliguria d. Neonatal macrosomia

ANS: d Feedback a. The patient is not especially high risk for a postpartum hemorrhage. b. The nurse would expect to see neonatal hypoglycemia, not hyperglycemia. c. The nurse would expect to see postpartum polyuria. d. The nurse would expect to see neonatal macrosomia.

10. The nurse working in a prenatal clinic is providing care to three primigravida patients. Which of the patient findings would the nurse highlight for the physician? a. 15 weeks, denies feeling fetal movement b. 20 weeks, fundal height at the umbilicus c. 25 weeks, complains of excess salivation d. 30 weeks, states that her vision is blurry

ANS: d Feedback a. This finding is normal. Quickening is usually felt between 16 and 20 weeks' gestation. b. This finding is normal. The fundal height at 20 weeks' gestation is usually at the level of the umbilicus. c. Excess salivation is a normal, albeit annoying, finding. d. Blurred vision is a sign of pregnancy-induced hypertension (PIH). This finding should be reported to the woman's health-care practitioner.

A nurse is caring for a woman 10 hours post-cesarean birth. She received a dose of intrathecal morphine at the time of the birth. Which of the following assessment data would require immediate intervention? a. Itching of the palms and feet b. Nausea c. Urinary output of 300 mL in the past 4 hours d. Respiratory rate of 10 breaths/minute

ANS: d Feedback a. This is a side effect of intrathecal morphine which is not life threatening. b. This is a side effect of intrathecal morphine which is not life threatening. c. A urinary output of 300 mL in 4 hours is within normal limits. d. Correct. An adverse effect of intrathecal morphine that requires immediate intervention is respiratory distress.

The NICU nurse is providing care to a 35-week-old infant who has been in the neonatal intensive care unit for the past 3 weeks. His mother wants to breastfeed her son naturally but is currently pumping her breasts to obtain milk. His mother is concerned that she is only producing about 1 ounce of milk every 3 hours. The nurse's best response to the patient's mother would be: a. "Pumping is hard work and you are doing very well. It is good to get about 1 ounce of milk every 3 hours." b. "Natural breastfeeding will be a challenging goal for your baby. Beginning today, you will need to begin to pump your breasts more often." c. "Your baby will not be ready to go home for at least another week. You can begin to pump more often in the next few days in preparation for taking your child home." d. "You have been working hard to give your son your breast milk. We can map out a schedule to help you begin today to pump more often to prepare to take your baby home."

ANS: d Feedback a. This is correct information but does not assist the women in producing more milk. b. This does not provide her with a plan to increase her milk. c. This does not provide her with a plan. d. The mother should be praised for her efforts to breastfeed and encouraged to continue to pump her milk. A determined schedule for pumping the milk will help the mother keep her milk flow steady and provide enough nutrients for the infant after discharge.

A nurse is admitting a woman for a scheduled cesarean section. Which of the following assessment data should be immediately reported to the physician? a. White cell count of 11,000 b. Hemoglobin of 11 g/dL c. Hematocrit of 33% d. Platelet count of 97,000

ANS: d Feedback a. This laboratory value is within normal limits for a pregnant woman. b. This laboratory value is within normal limits for a pregnant woman. c. This laboratory value is within normal limits for a pregnant woman. d. Normal range of platelets is 150,000 to 400,000. A low platelet count places the woman at risk for increased bleeding.

20. Which of the following statements is most appropriate for the nurse to say to a patient with a complete placenta previa? a. "During the second stage of labor you will need to bear down." b. "You should ambulate in the halls at least twice each day." c. "The doctor will likely induce your labor with oxytocin." d. "Please promptly report if you experience any bleeding or feel any back discomfort."

ANS: d Feedback a. This response is inappropriate. This patient will be delivered by cesarean section. b. This response is inappropriate. Patients with placenta previa are usually on bed rest. c. This response is inappropriate. This patient will be delivered by cesarean section. d. Labor often begins with back pain. Labor is contraindicated for a patient with complete placenta previa.

If the umbilical cord prolapses during labor, the nurse should immediately: a. Type and cross-match blood for an emergency transfusion. b. Await MD order for preparation for an emergency cesarean section. c. Attempt to reposition the cord above the presenting part. d. Apply manual pressure to the presenting part to relieve pressure on the cord.

ANS: d Feedback a. Type and cross-match is one of the interventions with cord prolapse but not a priority. b. Awaiting MD intervention is not appropriate as umbilical cord prolapse is an obstetrical emergency requiring immediate intervention. c. Once the cord has prolapsed, it cannot be repositioned. d. Apply manual pressure to the presenting part to relieve pressure on the cord represents the first nursing intervention to attempt to improve circulation to the fetus.

The nurse is massaging a boggy uterus. The uterus does not respond to the massage. Which medication would the nurse expect would be given first: a. Methergine b. Ergotrate c. Carboprost d. Oxytocin or pitocin

ANS: d If the cause of the hemorrhage is uterine atony, continual fundal massage with lower uterine segment support is mandatory. While one member of the team massages the fundus, another nurse establishes intravenous access with a large bore needle and administers oxytocic drugs in the following order: oxytocin (Pitocin), followed by methylergonovine (Methergine) or ergonovine (Ergotrate), and carboprost (Hemabate).

34. The single most important risk factor for preterm birth includes: a. Uterine and cervical anomalies b. Infection c. Increased BMI d. Prior preterm birth

ANS: d The single most important factor is prior preterm birth with a reoccurrence rate of up to 40%.

The clinic nurse sees Xiao and her infant in the clinic for their 2-week follow-up visit. Xiao appears to be tired, her clothes and hair appear unwashed, and she does not make eye contact with her infant. She is carrying her son in the infant carrier and when asked to put him on the examining table, she holds him away from her body. The clinic nurse's most appropriate question to ask would be: a. "What has happened to you?" b. "Do you have help at home?" c. "Is there anything wrong with your son?" d. "Would you tell me about the first few days at home?"

ANS: d The well-baby checkup that generally takes place 1 to 2 weeks following the hospital discharge may offer the first opportunity to assess the mother-baby dyad. In this setting, the nurse needs to be alert for subtle cues from the new mother, such as making negative comments about the baby or herself, ignoring the baby's or other children's needs, as well as the mother's physical appearance. In a private area, the nurse should take time to explore the new mother's feelings. A nonthreatening way to open the dialogue might be to say: "Tell me how the first few days at home have gone." This statement provides the new mother with an opportunity to share both positive and negative impressions.

The perinatal nurse knows that the presence of abdominal distension and gas in the post-cesarean birth mother is due to __________.

ANS: delayed peristalsis Delayed peristalsis and constipation commonly occur because of slowed peristalsis associated with pregnancy hormones and childbirth anesthesia. In addition, incisional pain may contribute to a decrease in ambulation which contributes to delayed peristalsis.

The perinatal nurse understands that the most appropriate nursing action following an amniotomy is an assessment of the __________ as well as the __________ and __________ of the amniotic fluid.

ANS: fetal heart rate; color; odor The nurse carefully monitors the patient who will undergo an amniotomy. Vital signs, cervical effacement and dilation, station of the presenting part, fetal heart rate, and color and amount of amniotic fluid are assessed.

The perinatal nurse prepares for two potential complications that may accompany a precipitous labor and birth: postpartum __________ and a need for neonatal __________.

ANS: hemorrhage; resuscitation Feedback 1: Complications from a precipitate labor pattern result from trauma to maternal tissue and to the fetus because of the rapid descent. Hemorrhage may occur from uterine rupture and vaginal lacerations. The fetus may suffer from hypoxia related to the decreased periods of uterine relaxation between the contractions and intracranial hemorrhage related to the rapid birth. Feedback 2: Complications from a precipitate labor pattern result from trauma to maternal tissue and to the fetus because of the rapid descent. Hemorrhage may occur from uterine rupture and vaginal lacerations. The fetus may suffer from hypoxia related to the decreased periods of uterine relaxation between the contractions and intracranial hemorrhage related to the rapid birth.

The perinatal nurse provides information about postpartum depression to all families members because of the potential danger not only to the mother but also to the __________.

ANS: infant The earlier that postpartum depression is recognized and treatment begun, the better is the prognosis for a full recovery. The nurse should involve the family in helping the patient cope with her feelings and assisting with infant care.

The perinatal nurse caring for a laboring woman who is receiving an oxytocin infusion documents the following information: rate of __________, frequency and strength of __________, fetal __________, and cervical __________ and __________.

ANS: infusion; contractions; heart rate; dilatation; effacement Oxytocin protocols in many institutions require that the nurse remain at the patient's bedside at all times for careful surveillance. The following data should be placed on a flow sheet in the patient record: patient's vital signs, fetal heart rate, frequency, duration and strength of contractions, cervical effacement and dilatation, fetal station and lie, rate of oxytocin infusion intake and urine output, and the psychological response of the patient.

During labor, oxytocin is always administered __________.

ANS: intravenously with an infusion pumpDuring labor, oxytocin can only be administered intravenously via an infusion pump to titrate and regulate the dose for safe administration.

30. Providing information to parents about jaundice constitutes an important component of the nurse's discharge teaching. Ensuring that parents know when and who to call if their infant develops signs of jaundice will help decrease the risk of __________, or permanent brain damage.

ANS: kernicterus All newborns are screened before discharge for physiological jaundice. The central nervous system can be damaged from unconjugated bilirubin. If bilirubin crosses the blood-brain barrier, it can damage the cerebrum, causing a condition called kernicterus. Kernicterus occurs from brain cell necrosis and can permanently damage a newborn, depending on the amount of time the neurons are exposed to bilirubin, the susceptibility of the nervous system, and the function of the surviving neurons.

Providing information to parents about jaundice constitutes an important component of the nurse's discharge teaching. Ensuring that parents know when and who to call if their infant develops signs of jaundice will help decrease the risk of __________, or permanent brain damage.

ANS: kernicterus All newborns are screened before discharge for physiological jaundice. The central nervous system can be damaged from unconjugated bilirubin. If bilirubin crosses the blood-brain barrier, it can damage the cerebrum, causing a condition called kernicterus. Kernicterus occurs from brain cell necrosis and can permanently damage a newborn, depending on the amount of time the neurons are exposed to bilirubin, the susceptibility of the nervous system, and the function of the surviving neurons.

A nurse assesses a G2 P1 woman who gave birth to a 4500 gram baby boy 2 hours ago. The nurse notes that the woman's labor was only 2 hours and that the infant was delivered by the labor nurse. The nurse's assessment findings are: Fundus firm and midline at umbilicus Lochia heavy—saturates pad within 15 minutes and bleeding is a steady stream without clots Perineum intact, slight bruising Ice pack on perineum Vital signs are B/P 105/65, P 98, R 20, T 38° Based on this information, the nurse is concerned that the woman has a __________ of the __________ or __________.

ANS: laceration; cervix; vagina Based on the assessment data, the woman is experiencing an early postpartum hemorrhage (PPH). The hemorrhage is most likely not due to uterine atony because the fundus is firm and midline. Laceration of the cervix or vagina is the second most common cause of early PPH. This woman is displaying typical signs and symptoms of laceration of cervix or vagina—firm, midline fundus with steady stream of blood without clots.

33. Part of the assessment of a preterm infant includes obtaining an abdominal girth measurement. The NICU nurse performs this assessment because the patient is at risk for __________.

ANS: necrotizing enterocolitis (NEC) When caring for a child with necrotizing enterocolitis, the nurse must measure and record frequent abdominal circumferences, auscultate bowel sounds before every feeding, and observe the abdomen for distention (observable loops or shiny skin indicating distention).

Part of the assessment of a preterm infant includes obtaining an abdominal girth measurement. The NICU nurse performs this assessment because the patient is at risk for __________.

ANS: necrotizing enterocolitis (NEC) When caring for a child with necrotizing enterocolitis, the nurse must measure and record frequent abdominal circumferences, auscultate bowel sounds before every feeding, and observe the abdomen for distention (observable loops or shiny skin indicating distention).

Postpartum woman are at an increased risk of thrombus formation immediately following birth due to an increased __________ level.

ANS: plasma fibrinogen Levels of plasma fibrinogen tend to remain elevated during the first few postpartal weeks. Although this alteration exerts a protective effect against hemorrhage, it increases the patient's risk of thrombus formation.

31. The NICU nurse recognizes that the infant who requires ventilation for meconium aspiration syndrome is most often __________.

ANS: post-term Meconium aspiration pneumonia occurs in 10% to 26% of all deliveries, and the incidence increases directly with gestational age. (Before 37 weeks' gestation there is a 2% incidence, and at 42 weeks' gestation there is a 44% incidence.)

The NICU nurse recognizes that the infant who requires ventilation for meconium aspiration syndrome is most often __________.

ANS: post-term Meconium aspiration pneumonia occurs in 10% to 26% of all deliveries, and the incidence increases directly with gestational age. (Before 37 weeks' gestation there is a 2% incidence, and at 42 weeks' gestation there is a 44% incidence.)

The perinatal nurse recognizes that the laboring multiparous patient who is attempting a vaginal birth following a previous cesarean birth (VBAC) needs frequent assessments to ensure that there is __________ during her labor.

ANS: progress Women with a previous history of cesarean birth may be offered a trial of labor, although a prompt cesarean birth is recommended at the earliest sign of maternal or fetal compromise.

A postpartum woman who describes symptoms of hallucinations and suicidal thoughts is most likely experiencing postpartum __________.

ANS: psychosis Postpartum psychosis is a rare but severe form of mental illness that severely affects not only the new mother, but the entire family. Postpartum psychosis may present with symptoms of postpartum depression. However, the distinguishing signs of psychosis are hallucinations, delusions, agitation, confusion, disorientation, sleep disturbances, suicidal and homicidal thoughts, and a loss of touch with reality.

The development of a large hematoma can place the postpartum woman at risk for __________.

ANS: shock Upon examination of the perineal or vulvar areas, the nurse may notice discoloration and bulging of the tissue at the hematoma site. If touched, the patient complains of severe tenderness, and the clinician generally describes the tissue as "full." If the hematoma is large, signs of shock may be evident, and the patient may exhibit an absence of lochia and an inability to void.

The Joint Commission Standard states that the __________, __________, and __________ are accurately identified and clearly communicated during the final verification process before the start of any surgical or invasive procedure.

ANS: site; procedure; patient To decrease the risk of surgery or invasive procedure being done on the wrong patient or in the wrong site, a "time-out" is called, and active communication to verify correct procedure, site, and patient is done just prior to the beginning of surgery or invasive procedure.

The perinatal nurse explains to a new mother that the first sign of a postpartum infection will most likely be an increased __________.

ANS: temperature During the immediate postpartum period, the most common site of infection is the uterine endometrium. This infection presents with a temperature elevation over 101°F (38.4°C), often within the first 24 to 48 hours after childbirth, followed by uterine tenderness and foul-smelling lochia.

32. The NICU nurse's patient assignment includes an infant who is 25 weeks' gestation. The nurse knows that according to the gestational age, this infant would be described as __________.

ANS: very premature The definition of very premature is a neonate born at less than 32 weeks' gestation. The definition of premature is a neonate born between 32 and 34 weeks' gestation. The definition of late premature is a neonate born between 34 and 37 weeks' gestation.

It is critical for the perinatal nurse to learn, as part of the facility's policies and procedures, to immediately perform a vaginal examination on a woman who presents with vaginal bleeding after twenty-four weeks gestation. A) True B) False

B) False

The perinatal nurse knows that the most common chromosomal pattern found in a complete molar pregnancy is 32XX due to the absence of maternal genetic material. A) True B) False

B) False

The clinic nurse assesses Marijke, a 30-year old primigravid woman at 15 weeks gestation. Marijke's blood pressure, measured twice at intervals one hour apart, is 146/96. This finding is best described as: A) Preeclampsia B) Pregnancy-induced hypertension C) Chronic hypertension D) Transient hypertension

C) Chronic hypertension

The perinatal nurse knows that tocolytic agents are most often used to: Select all answers that apply: A) Prevent maternal infection B) Prolong pregnancy as long as possible C) Facilitate administration of antenatal corticosteroids D) Allow for transport of the woman to a tertiary care facility

C) Facilitate administration of antenatal corticosteroids D) Allow for transport of the woman to a tertiary care facility

Karen, a 22-year-old woman with vaginal bleeding, has come to the triage unit for assessment and pain medication. She has missed one period and, following a transvaginal ultrasound, her pregnancy is confirmed. However, implantation has occurred in the right fallopian tube. The ectopic mass is 3 cm and has not ruptured. The most appropriate therapy would be: A) Partial salpingectomy B) Laparoscopic salpingostomy C) Methotrexate D) Salpingectomy by laparotomy

C) Methotrexate

Placement of ligature to close the cervix

Cervical cerclage

Invasive, malignant trophoblastic disease that is usually metastatic and can be fatal

Choriocarcinoma

The perinatal nurse is assessing a woman in triage who is 34 + 3 weeks gestation in her first pregnancy. She is worried about having her baby "too soon" and she is experiencing uterine contractions every 10-15 minutes. The fetal heart rate is 136 beats per minute. A vaginal examination performed by the health care provider reveals that that the cervix is closed, long, and posterior. The most likely diagnosis would be: A) Preterm labor B) Term labor C) Back labor D) Braxton-Hicks contractions

D) Braxton-Hicks contractions

The perinatal nurse knows that the term to describe a woman at 26 weeks gestation with a history of elevated blood pressure who presents with a urine showing 2+ protein (by dipstick) is: A) Preeclampsia B) Chronic hypertension C) Gestational hypertension D) Chronic hypertension with superimposed preeclampsia

D) Chronic hypertension with superimposed preeclampsia

A disease characterized by an abnormal placental development that results in the production of fluid-filled grapelike clusters and a vast proliferation of trophoblastic tissue

Gestational trophoblastic disease

No expulsion of the products of conception, but bleeding and dilation of the cervix such that a pregnancy is unlikely

Inevitable abortion

A pregnancy that ends before 20 weeks gestation

Miscarriage

A biochemical marker being investigated for its predictive value in preterm labor

Salivary estriol

Specks or spots in the vision where the patient cannot see; "blind spots"

Scotomata

A condition where the umbilical cord is implanted into the membranes rather than the placenta

Vasa previa


संबंधित स्टडी सेट्स

Chapter 10- T test for related samples

View Set

Gateway to Business- Accounting Pop Quiz

View Set

Biology: Molecular Basis of Life

View Set

A & P 2 - Chapter 23 - The Urinary System

View Set

Chapter 10 Abnormal psychology corrections

View Set

1-10 Distributive Property of Multiplication (2x6) + (2 x 6) = 4 x 6

View Set

психованная с 151 по 300 вопрос

View Set

Maternal Child Nursing Care Chapter 10 Assessment of High Risk Pregnancy

View Set

8. ECS, Lambda, Batch, LightSail

View Set

Diagnostic Techniques in Counseling - Chapter 1

View Set